Đến nội dung

Ban Biên Tập

Ban Biên Tập

Đăng ký: 01-01-2012
Offline Đăng nhập: 01-08-2013 - 22:35
***--

#303262 Số đầu mối tối thiểu trong trò chơi Sodoku

Gửi bởi Ban Biên Tập trong 10-03-2012 - 00:29

Đối với các nhà toán học, đột nhiên được giải quyết một vấn đề lâu dài luôn là một khoảnh khắc hạnh phúc. Năm 2012 bắt đầu với thời điểm như vậy, khi một nhà toán học Ireland tên là Gary McGuire đã công bố một giải pháp cho vấn đề số đầu mối tối thiểu trong trò chơi Sudoku.

Bạn đã thấy trò chơi Sudoku? không nghi ngờ gì nữa, vì ngày nay, nó phổ biến trên các báo và tạp chí. Trông nó như thế này:

Hình đã gửi

Nhiệm vụ của bạn là điền vào trong các ô bỏ trống với những chữ số từ 1-9 theo cách như vậy mỗi hàng, cột và khối 3x3 chứa mỗi con số chính xác một lần. Trong một trò chơi thích hợp, các đầu mối bắt đầu như vậy để đảm bảo có 1 và chỉ một cách để hoàn thành hình vuông.

Trò chơi này đặc biệt chỉ có 17 đầu mối bắt đầu. Nó đã từ lâu được tin rằng 17 là số lượng tối thiểu cho bất kỳ 1 trò chơi thích hợp. Nhà toán học Gordon Royle duy trì một cơ sở dữ liệu trực tuyến hiện có gần 50.000 câu đố với 17 đầu mối bắt đầu (trong thực tế, các câu đố trên được chuyển thể từ một trong những câu đố trong danh sách đó). Tuy nhiên, mặc dù tìm kiếm bằng máy tính, người ta vẫn không tìm thấy ví dụ cho 1 trò chơi với 16 hoặc ít hơn những đầu mối.

Vấn đề là tìm kiếm máy tính đầy đủ dường như không thể. Đơn giản bởi có quá nhiều khả năng để xem xét. Ngay cả bằng cách sử dụng các phần cứng hiện đại nhất, và sử dụng các kỹ thuật tìm kiếm hiệu quả nhất được biết đến, hàng trăm hàng ngàn năm mới được yêu cầu.

Tương tự, Toán học thuần túy cũng hỗ trợ được rất ít. Nó rất dễ dàng để thấy rằng bảy đầu mối là không đủ. Với bảy đầu mối bắt đầu sẽ có ít nhất hai chữ số mà không được xuất hiện khi bắt đầu trò chơi. Để cụ thể, chúng ta hãy nói rằng đã có không có số 1 hoặc 2 trong bảng bắt đầu. Khi đó, trong bất kỳ cách hoàn thành nào, chỉ cần thay các ô 1 bởi 2 và các ô 2 bởi 1, ta sẽ tạo ra một lời giải thứ hai hợp lệ để câu đố. Tuy nhiên, sau khi quan sát này, ta đã không biết làm thế nào để tiếp tục. Ngay cả một lập luận đơn giản là chứng minh sự bất cập của tám đầu mối đã được khẳng định là rất khó

Giải pháp của McGuire đòi hỏi một sự kết hợp của toán học và khoa học máy tính. Để giảm thời gian cần thiết cho một tìm kiếm đầy đủ, ông đưa ra ý tưởng về một "tập hợp không thể tránh khỏi" (Hãy xét các ô được tô đậm)

Hình đã gửi

Bây giờ tưởng tượng ra một câu đố bắt đầu có hình vuông này cho một giải pháp. Bạn có thể thấy lý do tại sao chúng ta sẽ cần phải có ít nhất một đầu mối bắt đầu từ một trong những “tế bào” được tô đậm? Lý do là nếu không như vậy, sau đó ta sẽ có thể chuyển đổi các chữ số trong các ô để tạo ra một giải pháp thứ hai cho cùng một câu đố. Trong thực tế, đặc biệt Sudoku vuông, có rất nhiều bộ tương tự như "bộ không thể tránh khỏi", nói chung một số hình vuông sẽ có nhiều hơn những cái khác, và các loại khác nhau. Một phần của giải pháp McGuire liên quan đến việc tìm kiếm một bộ sưu tập lớn của một số "các bộ không thể tránh khỏi" trong mỗi hình vuông Sudoku được xem xét.

Tìm kiếm các "bộ không thể tránh khỏi" cho phép giảm đáng kể kích thước của không gian phải được tìm kiếm. Thay vì tìm kiếm thông qua tất cả các tập hợp con 16 đầu mối của một Sudoku vuông, sự tìm kiếm tuyệt vọng thực sự cho một câu đố thích hợp, chúng tôi chỉ cần xem xét tập hợp của 16 đầu mối bắt đầu có chứa ít nhất một “tế bào” từ mỗi “bộ không thể tránh khỏi” được biết đến. Tìm kiếm những bộ cụ thể của các đầu mối bắt đầu là 1 ví dụ cụ thể của 1 vấn đề tổng quát hơn, đó là thuật toán để giải quyết các vấn đề tập hợp đánh một lượng hợp lý "vấn đề tập hợp đánh." số lần. Giải quyết vấn đề tối thiểu đầu mối cho Sudoku là một ứng dụng của thuật toán mới này.

Tất nhiên, thận trọng là cần thiết cho đến khi các nhà nghiên cứu có thời gian để kiểm tra kỹ các lập luận của McGuire. Nó là một trong những sự “tàn bạo” của toán học nhằm cẩn thận tránh những lỗi tinh tế của các nghiên cứu viên. Chúng ta chắc chắn có thể nói, mặc dù, rằng các kỹ thuật đang được sử dụng ở đây là rất chính đáng và thú vị. Họ cũng có thể là hữu ích cho việc đánh bóng "đầu mối tối thiểu" các vấn đề trong Sudoku. Ví dụ, nếu chúng tôi yêu cầu rằng các đầu mối bắt đầu được đặt đối xứng, sau đó 18 đầu mối là nhỏ nhất được biết đến (như được hiển thị bên dưới bên trái, câu đố lấy từ Taking Sudoku Seriously). Mặt khác, Sudoku X, trong đó các 2 chính đường chéo tham gia các hàng, cột và các khối là khu vực hợp lệ Sudoku, các tối thiểu hiện nay là 12 (xem bên dưới phải; câu đố phù hợp từ một bộ sưu tập trực tuyến Ruud của 12-đầu mối Sudoku X).

Hình đã gửi


Hình đã gửi
Sodoku X


Bài báo của McGuire minh họa hai trong số những thực tế của toán học hiện đại. Đầu tiên là vai trò nổi bật của máy tính. Ngày nay nó là ngày càng khó khăn để xác định câu đố nên được xem như các vấn đề toán học, và thực sự là vấn đề cho các nhà khoa học máy tính.


Tuy nhiên, thứ hai là tốc độ phát hiện toán học. Chúng ta đã học được của công việc McGuire như chúng ta đã đi đến một hội nghị toán học mà chúng tôi đã tổ chức một phiên họp về các câu đố Sudoku. Một người trong chúng ta đã có một cuộc nói chuyện chuẩn bị, trong đó vấn đề này được mô tả là chưa được giải quyết. Nó rất thú vị để có thể sử dụng hội nghị để báo cáo về sự phát triển này. Cuốn sách về toán học của Sudoku của chúng ta đã chỉ ra trong một vài tuần, nhưng nó có thể đã bị lỗi thời, ít nhất là đối với một vấn đề này.

Hoàng Ngọc Thế dịch từ http://blog.oup.com




#302877 GS Ngô Bảo Châu viết 'tiểu thuyết toán hiệp'

Gửi bởi Ban Biên Tập trong 08-03-2012 - 12:51

Tôi may mắn có trong tay bản thảo cuốn sách đã ba tháng nay, và phải làm cái công việc khó khăn là không dám “khoe” vì sợ lộ bí mật. Nay cuốn sách đã sắp ra mắt, xin đưa lên bài viết về cảm nhận của tôi khi đọc sách. Cũng có thể xem là Lời giới thiệu cuốn sách với bạn đọc (của trang blog này).

Một cuốn sách trình bày những kiến thức toán học dưới dạng “ngây thơ” nhất, và vì thế, bản chất nhất. Ta gặp ở đây những khái niệm đầu tiên về số, những vấn đề “cao siêu” như trường số, tính không giải được bằng căn thức của phương trình bậc 5 tổng quát, cho đến hình học phi Euclid, đường trắc địa…Một cuốn lịch sử toán học, từ sự ra đời của số vô tỷ đầu tiên đến sự xuất hiện của phương pháp tọa độ và giải tích vô cùng bé. Nhưng trước tiên, cuốn sách là một câu chuyện cổ tích tuyệt đẹp, với nắng vàng trên bãi cát, với bầu không khí trong vắt của trí tuệ và tình người, với những nghịch lý và âm mưu. Ta chợt nhận ra rằng, thế giới của những con số cũng huyền ảo, kỳ bí và lãng mạn như cuộc đời.

Tác giả đưa ta vào câu chuyện cổ tích, mà ở đó cùng với Ai và Ky, ta bắt gặp những con người của nhiều thế kỷ khác nhau đang sống cùng nhau trong một thế giới chung: Toán học. Ta gặp Euclid (thế kỷ III TCN) và Thales (624-546 TCN), hai nhà phù thủy chỉ dùng cây thước và compa mà dựng nên cả “thế giới hữu tỷ”; thấy Pythagoras (570-495 TCN) nghiêm khắc trừng phạt anh học trò Hyppasus (thế kỷ V TCN) vì đã dám tạo ra số vô tỷ đầu tiên, phá vỡ cài hài hòa của thế giới hữu tỷ. Ta gặp Zeno xứ Elea (490- 430 TCN) hiện thân thành cô gái Zena với cặp mắt mê hoặc, mà khi nhìn vào đó, thời gian dường như bị chia thành những mảnh nhỏ vô cùng mà ta không thể vượt qua. Nhưng cuối cùng, Alice (lực sĩ Achilles?) đã dũng cảm vượt qua cái “nghịch lý Zeno” đó, dù biết là phải chịu trừng phạt. Để vượt qua cái việc chia cắt thời gian thành những mảnh nhỏ, Alice phải nhờ tới “chuỗi hội tụ”, nhờ tới giải tích vô cùng bé. Ông Chico hiểu điều đó, nhưng vẫn trừng trị Alice, để không làm phật ý Đức Vua, người chỉ biết dùng phép khai căn để tìm ra gốc gác từng thần dân (số) của mình. Ta nhận ra trong bóng dáng Chico hình ảnh của Cauchy (1789-1857), nhà toán học thiên tài, một trong những người đặt nền móng cho giải tích vô cùng bé, nhưng cũng là một người bảo hoàng cực đoan. Cũng vì phụng sự Đức Vua mà Chico trừng phạt chàng thanh niên Elaci, người đã phát hiện ra rằng Đức Vua không thể chỉ dùng phép khai căn mà tìm được những “thần dân” là nghiệm của phương trình bậc năm. Ông Chico muốn dìm đi phát hiện đó của chàng Elaci, và Elaci đã phải quyết đấu với con rồng. Nhưng trong câu chuyện cổ này, Elaci không chết ở tuổi 21 như Evariste Galois (1811-1832) trong buổi sáng định mệnh sau trò đấu súng, mà chỉ bị đày đến “xứ p-adic”, xứ sở mới của những con số khác với số thực và phức nhưng cũng không kém phần hấp dẫn.

Ai và Ky ở xứ sở của những con số tàng hình còn là cuốn sách vỡ lòng về triết học của toán học. Với cặp mắt ngây thơ của Ai, ta nhìn ra mâu thuẫn ẩn chứa trong các chứng minh phản chứng, mâu thuẫn giữa cái hữu hạn và vô hạn, của sự chứng minh được hay không một chân lý toán học. Bởi thế nên ông già Cartesius (René Descartes, 1596-1650) mới nhận ra rằng, để “tồn tại” thì chỉ “suy nghĩ” thôi là chưa đủ: “Ta đang nghi ngờ, việc của ta là nghi ngờ. Ta sợ một lúc nào đó ta không còn nghi ngờ được nữa, thì bản thân ta cũng sẽ trở nên tàng hình”. Cũng bởi vì thế mà Diogenes xứ Sinop (412 hoặc 404 – 323 TCN) mới cầm đèn đi giữa ban ngày để tìm cho ra người lương thiện.

“Giải quyết một bài toán cũng như ăn từng miếng nhỏ của trái táo. Làm như thế một là để thưởng thức, hai là để tránh nghẹn”, ông già Cartesius đã khuyên Ai như thế. Cuốn sách trên tay các bạn cũng cần được thưởng thức chậm rãi như thế, để có thể cảm nhận được hương vị đậm đà của nó. Và trên hết, phải có cặp mắt ngây thơ như cậu thiếu niên Ai, để nhìn ra chân lý. Bởi vì chân lý chỉ dành cho những người không bị che mắt bởi cái vụn vặt thường nhật.

Lời nhắn nhủ của Elaci từ xứ sở p-adic cũng là lời nhắn cho tất cả chúng ta: “Hãy lắng nghe những người khác nói nhưng luôn suy nghĩ bằng cái đầu của mình. Cuối cùng và quan trọng nhất là đừng bao giờ để rơi mất tiếng cười hồn nhiên của Alice”

Cuốn sách giản dị, thông thái, sâu sắc, vừa dễ gần, vừa không dễ hiểu - như chính tác giả của nó vậy.

Gấp sách lại, cái “cảm giác tiếc nuối trở nên mơ hồ như làn khói lam chiều. Có một tiểu hành tinh vừa băng qua mùa thu của trái đất”.


#302782 Kovalevskaya, Một số phận vinh quang và cay đắng

Gửi bởi Ban Biên Tập trong 07-03-2012 - 20:44

Nhân ngày quốc tế phụ nữ 08/03, BBT xin giới thiệu với các bạn tiểu sử nhà nữ toán học xuất sắc nhất.

Sofia Vasilyevna Kovalevskaya (15 tháng 1 [cũ 3 tháng 1] năm 1850 – 10 tháng 2 [cũ 29 tháng 1] năm 1891), là nhà toán học lớn của Nga, với nhiều đóng góp quan trọng cho các ngành thống kê, phương trình vi phân và cơ học, và là người phụ nữ đầu tiên được trao học hàm giáo sư toàn diện ở Bắc Âu.

Có một số cách chuyển tự tên của bà. Bà thường hay sử dụng tên Sophie Kowalevski (hoặc thỉnh thoảng Kowalevsky), khi xuất bản khoa học. Sau khi chuyển sang sống tại Thụy Điển, bà gọi mình là Sonya.

Thơ ấu

Sofia Kovalevskaya (nhũ danh Korvin-Krukovskaya), sinh tại Moskva, là con thứ hai trong gia đình có 3 người con. Cha của bà, Vasily Vasilyevich Korvin-Krukovsky, là Trung tướng Pháo binh phục vụ trong Quân đội Đế quốc Nga. Mẹ của bà, Yelizaveta Fedorovna Schubert, là một phụ nữ có học thức thuộc gốc Đức còn bà của Sofia là người Rumani. Họ là những người thắp lên niềm say mê toán học cho bà và mướn cho bà một gia sư, (A. N. Strannoliubskii , một nhân vật nổi tiếng về ủng hộ quyền học cao của phụ nữ) để dạy cho bà môn số học. Cũng trong thời gian đó, một đứa con trai của một tu sĩ trong vùng đã giới thiệu thuyết hư vô cho bà.[2]

Tuy tài năng toán học của bà thể hiện khá rõ, bà không thể hoàn tất việc học hành tại Nga. Vào thời đó, phụ nữ không được phép đi học đại học. Để được đi học ở nước ngoài, bà cần thư viết tay của cha (hoặc chồng). Vì vậy, bà đã dựng nên một cuộc "kết hôn giả" với Vladimir Kovalevsky, người đang là sinh viên ngành cổ sinh vật học, sau này nổi tiếng khi cộng tác với Charles Darwin. Họ rời khỏi nước Nga vào năm 1867[3].

Thời sinh viên

Năm 1869, Kovalevskaya nhập học trường Đại học Heidelberg, Đức, nơi cho phép bà được dự thính các lớp miễn là được sự cho phép của giáo sư dạy lớp đó.

Một thời gian ngắn sau đó, bà đến Luân Đôn cùng với Vladimir, do Vladimir có công việc hợp tác cùng Thomas Huxley Charles Darwin, còn bà thì được mời tham gia vào những cuộc mạn đàm ngày Chủ nhật của George Eliot. Tại đó, vào tuổi 19, bà gặp Herbert Spencer và được Eliot dẫn dắt vào một cuộc tranh luận về "khả năng tư duy trừu tượng của phụ nữ".

Sau hai năm học toán tại Heidelberg cùng với các thầy giáo như Helmholtz, Kirchoff Bunsen, bà chuyển đến Berlin, và đi học riêng với Karl Weierstrass, vì trường đại học ở đây không cho phép bà đi học cả dự thính. Vào năm 1874 bà trình ba bài báo—về phương trình vi phân bán phần, về sự chuyển động của vành sao thổ và tích phân ê-líp—cho Đại học Göttingen xem như là luận văn tiến sĩ của mình. Với sự ủng hộ của Weierstrass, bà tốt nghiệp tiến sĩ toán học với loại xuất sắc, bỏ qua các môn học và những kỳ thi bắt buộc. Từ đó bà trở thành người nữ đầu tiên ở châu Âu có được học vị này. Bài báo của bà về phương trình vi phân bán phần có mô tả một định lý mà ngày nay được biết đến với tên Định lý Cauchy-Kovalevski, nêu ra những điều kiện đẻ xác định sự tồn tại lời giải cho một nhóm phương trình đó.

Năm cuối đời ở Đức và Thụy Điển


Gia đình Kovalevsky trở lại Nga, nhưng không được công nhận chức danh giáo sư vì chính kiến có phần cấp tiến của họ. Thất vọng, họ quay trở lại Đức. Vladimir, người thường xuyên bị chứng thay đổi cảm xúc, dần trở nên không ổn định nên họ sống cách ly trong phần lớn thời gian. Sau đó, vì một lý do nào đó, họ quyết định sống vài năm với nhau như vợ chồng thật sự. Trong thời gian này họ có được một đứa con gái Sofia (còn gọi là "Fufa"). Sau một năm dành thời gian để nuôi nấng con, Kovalevskaya gửi Fufa cho chị gái chăm sóc để tiếp tục nghiên cứu toán học và rời bỏ Vladimir. Vào năm 1883, với chứng thay đổi cảm xúc ngày càng tệ và có nguy cơ bị kiện vì lừa đảo chứng khoán, Vladimir đã tự tử.

Vào năm đó, với sự trợ giúp của nhà toán học Gösta Mittag-Leffler, trước đây cùng là sinh viên của Weierstrass, Kovalevskaya được trao học hàm privat-docent (một dạng phó giáo sư không ăn lương) tại Đại học Stockholm ở Thụy Điển.

Một năm sau đó (1884) bà được chỉ định vào vị trí "Professor Extraordinarius" (giáo sư không có ghế) và trở thành biên tập viên của Acta Mathematica. Vào năm 1888 bà giành giải Prix Bordin của Viện hàn lâm Khoa học Pháp, vì công trình "Mémoire sur un cas particulier du problème de le rotation d'un corps pesant autour d'un point fixe, où l'intégration s'effectue à l'aide des fonctions ultraelliptiques du temps" của bà. Bài báo này có một phát hiện nổi tiếng ngày nay được biết đến với tên "Đỉnh Kovalevskaya".

Vào năm 1889 bà được chỉ định làm "Professor Ordinarius" (giáo sư có ghế) tại Đại học Stockholm, trở thành người phụ nữ đầu tiên nắm giữ vị trí này tại một trường đại học ở Bắc Âu. Sau khi được tích cực vận động (và có sự thay đổi về điều lệ của Viện hàn lâm) bà được trao ghế Chủ tịch trong Viện hàn lâm Khoa học Nga, nhưng chưa bao giờ được chấp nhận hàm giáo sư tại Nga.

Kovalevskaya cũng viết một vài công trình không liên quan đến toán học, như cuốn hồi ký, Thời thơ ấu ở Nga, kịch (cộng tác với Nữ bá tước Anne Charlotte Edgren-Leffler) và một tiểu thuyết dạng hồi ký, Cô gái theo thuyết hư vô (1890).

Bà chết vì bệnh cúm vào năm 1891 vào tuổi 41, khi vừa mới trở về từ chuyến đi đến Genoa. Bà được chôn cất tại Solna, Thụy Điển, tại Norra begravningsplatsen
Việt Nam chúng ta còn biết đến bà qua 1 giải thưởng. Giải thưởng Kovalevskaya (còn gọi tắt là Giải "Kova") là giải thưởng thường niên dành tặng cho những nữ khoa học gia xuất sắc, nhằm tôn vinh những tập thể, cá nhân là các nữ khoa học có thành tích xuất sắc trong nghiên cứu và ứng dụng khoa học vào thực tiễn cuộc sống, đem lại nhiều lợi ích trên các lĩnh vực - kinh tế, xã hội và văn hóa.

Quỹ giải thưởng quốc tế Kovalevskaya được thành lập năm 1985, với sự đóng góp ban đầu của hai vợ chồng giáo sư người Mỹ AnnNeal Koblitz. Từ năm đó, vợ chồng giáo sư đã chọn các nhà khoa học nữ Việt Nam về lĩnh vực khoa học tự nhiên làm đối tượng để xét và trao Giải thưởngKovalevskaya. Quỹ đã hỗ trợ cho 8 nước đang phát triển là: Peru, El Salvador, Nicaragua, Mexico, Cuba, Nam Phi, Mozambic và Việt Nam.

Cho đến năm 2009, giải thưởng đã được trao cho 31 cá nhân và 15 tập thể các nhà khoa học nữ xuất sắc, tiêu biểu trong lĩnh vực khoa học tự nhiên.

Hiện nay, Lễ trao Giải thưởng này do Hội Liên hiệp Phụ nữ Việt Nam thực hiện. Bà Nguyễn Thị Bình, nguyên Phó chủ tịch nước Việt Nam là chủ tịch Ủy ban giải thưởng Kovalevskaya Việt Nam.
BBT mong rằng, Việt Nam nói chung và VMF nói riêng sẽ có nhiêu phụ nữ như Kovalevskaya.
Mời bạn thảo luận tại:


#302599 Topo là gì

Gửi bởi Ban Biên Tập trong 06-03-2012 - 22:33

Đi nghỉ hè về ít lâu rồi, nay lại viết một chút cho vui. Sắp tới tôi sẽ dạy con tôi về tô pô (topology). Câu hỏi đầu tiên là tô pô là gì, để làm gì ?

Tôi nhớ, hồi học đại học, tôi theo học chuyên ngành tô pô – hình học, nhưng cũng chưa bao giờ được thầy nào đề cập đến câu hỏi “tô pô là gì”. Cứ học một đống khái niệm về tô pô, và nghiễm nhiên chấp nhận rằng tô pô là môn học về những khái niệm trừu tượng đó (đóng, mở, compact, liên thông, đồng luân, đồng điều, v.v.). Cũng là kiểu “học gạo” thôi, nhồi đi nhồi lại vào đầu lâu ngày thành quen. Nhưng có lẽ phải mất rất lâu sau mới tự hiểu ra tô pô là gì, và giá như khi còn đi học được thày giải thích ngọn nguồn cho biết tô pô là gì và vì sao lại cần học tô pô, thì có lẽ tốt hơn.

Đối với phần lớn mọi người, thì từ “tô pô” là một từ khá xa lạ. Tôi còn nhớ, có lần có một cậu bạn trẻ hơn tôi một ít hỏi tôi học gì, tôi bảo học tô pô, cậu này liền nói là “anh bịa, làm gì có môn gì gọi là tô pô”. Trong quan điểm của “người thường”, toán học chỉ gồm có số học, rồi đến đại số, hình học, giải tích, còn “tô pô” là “bịa” rồi. Ngay các sinh viên hay nghiên cứu sinh ngành toán ở VN, nếu không theo học chuyên ngành hình học thì chắc cũng ít ai biết về tô pô.

Vậy thì tô pô là gì ?

Nói một cách đơn giản, tô pô (topology) chẳng qua là giải tích định tính (qualitative analysis). Cụm từ qualitative analysis chắc dễ hình dung hơn nhiều so với từ topology, bởi vì phần lớn mọi người (nếu đã qua đại học) có biết ít nhiều về giải tích là gì.

Thế nào là định tính ? Chẳng hạn khi ta nói “cái ô tô màu xanh lá cây”, thì “xanh lá cây” ở đây là định tính. Nếu nói đó là màu #008B45 theo bảng màu RGB (red-green-blue) thì đấy cũng là một màu xanh lá cây nhưng đã được định lượng chính xác hơn so với chỉ là “xanh lá cây”. Định tính là để nói về các tính chất chung, đối ngược với định lượng là để nói về các con số cụ thể chi tiết. Khi nói cái quả táo này to, thì đấy là định tính, còn khi nói nó nặng 425gr, thì đó là định lượng. Khi nói “phương trình này có nghiệm” thì đó là định tính, còn viết cụ thể ra nghiệm thì thành định lượng. Khi nói “đây là một đa tạp compact 3 chiều” thì các từ “đa tạp”, “compact”, “3 chiều” là các khái niệm tô pô, vì chúng là định tính, còn khi viết cụ thể ra phương trình của cái đa tạp đó trong không gian R4 chẳng hạn, thì đã thành hình học có tính định lượng hơn.

Từ topology có gốc Hy Lạp. Topo có nghĩa là “chỗ, vùng, miền, …” còn “logy” có nghĩa là “nghiên cứu, tìm hiểu, …”. Từ này được nhà toán học Hausdorff đưa ra vào năm 1914. Còn trước đó, tô pô được biết đến dưới tên gọi “analysis situs” (từ “situs” cũng có nghĩa là “chỗ, vùng, …”, nhưng là gốc Latin). Vào thập kỷ cuối của thế kỷ 19, Henri Poincaré có viết một loạt bài báo nhan đề “Analysis Situs”, trong đó ông đưa ra các khái niệm nền tảng cho tô pô, như đồng luân (homotopy), đồng điều (homology), đối ngẫu Poincaré, v.v. Vì có các công cụ đại số được đưa vào để nghiên cứu tô pô bắt đầu từ thời Poincaré, nên có một chuyên ngành toán hiện đại gọi là tô pô đại số. Cụm từ “analysis situs” được Leibniz dùng từ thời thế kỷ 17 để chỉ “hình học của các tình huống” (geometry of situations), với ý tưởng là đưa ra một ngôn ngữ hình học trừu tượng chung để có thể dùng để mô tả [một cách định tính ?] nhiều tình huống khác nhau. Dần dần cụm từ đó trở thành có nghĩa là tô pô, và ngày nay thì tô pô và hình học thường được coi là đi đôi với nhau. Có một tạp chí toán quốc tế nổi tiếng mang tên “Geometry and Topology”.

Các bài toán tồn tại hay không tồn tại những thứ gì đó trong toán học (ví dụ như tồn tại nghiệm của phương trình, điểm cực đại của hàm số, điểm bất động của ánh xạ, đồng phôi giữa hai đa tạp, v.v.) nhiều khi có thể coi là những bài toán có tính tô pô, và do vậy có thể sử dụng các công cụ của tô pô để hỗ trợ trong việc giải chúng. Một trong những bài toán rất cố điển dạng như vậy, mà có thể dùng làm bài toán đố cho trẻ em, là bài “3 cái nhà 3 cái giếng”: có 3 cái nhà và 3 cái giếng trên một mảnh đất, hỏi có thể hay không xây đường từ các nhà đến các giếng sao cho không có đường nào cắt (hay bắc cầu qua) đường nào. Bài toán “7 cái cầu ở Konigsberg” của Euler cũng thường được trích dẫn như là tiền sử của tô pô.

Việc hiểu biết các khái niệm về tô pô giúp chúng ta nhìn nhận nhiều vấn đề khác (có tính chất định tính) trong toán học một cách sáng sủa hơn. Một ví dụ là định lý arbitrage (kinh doanh chênh lệch giá) của toán tài chính (gần như tương đương với định lý đối ngẫu của qui hoạch tuyến tính), có thể dùng các tính chất của tập compact (là một khái niệm tô pô, và tính chất là hàm liên tục trên tập compact thì luôn có điểm cực đại) để chứng minh định lý này. (Tôi có viết cách chứng minh này trong sách toán tài chính với GS Đỗ Đức Thái). Ngày nay, các vấn đề của giải tích và hình học hiện đại hầu như luôn đụng đến các khái niệm tô pô.

Một số khái niệm cơ bản nhất của tô pô.

Một đặc điểm thường có của các tính chất định tính là khi ta thay đổi cái gì đó đi một chút ít thì tính chất của nó vẫn còn vậy. Chẳng hạn, một anh thuộc diện “nhà giàu”, nếu một hôm chẳng may mất đi 1 nghìn USD, thì anh ta vẫn còn là nhà giàu chứ không thành nhà nghèo ngay được. Trong tô pô, hiện tượng này được thể hiện bằng khái niệm lân cận (neighborhood), là một trong những khái niệm cơ bản nhất của tô pô. Nói một cách trực giác, một cái gì đó (trong ngôn ngữ hình học thì gọi là một điểm A) được gọi là nằm trong một lân cận của một điểm B nếu A “đủ gần” B sao cho A có cùng tính chất nào đó tương tự như B. Tất nhiên, mỗi điểm có thể có nhiều lân cận khác nhau, tùy theo coi thế nào là “gần”. Ví dụ, cả nước Pháp là một lân cận của tôi, vì ai ở nước Pháp cũng cùng chung tính chất “ở Pháp” với tôi. Toulouse là một lân cận khác của tôi (năm trong lân cận “Pháp”), cái dãy phố của tôi lại là lân cận nhỏ hơn nữa, còn “các nhà của người Việt ở Toulouse” sẽ là một lân cận khác nữa, v.v.

Tất nhiên, toán học cần ngôn ngữ chặt chẽ chính xác. Bởi vậy các lân cận phải thỏa mãn một hệ tiên đề nào đó của tô pô. Một tập hợp mà trong đó các lân cận của các phần tử thỏa mãn hệ tiên đề đó, thì được gọi là không gian tô pô.

Song song với khái niệm lân cận là khái niệm tập mở và tập đóng. Tiếp đến là các khái niệm: ánh xạ liên tục, tô pô cảm sinh, tô pô yếu hơn, tô pô mạnh hơn, đồng phôi, liên thông, liên thông đường, tính tách được, tính compact, tô pô của không gian metric, tương đương tô pô của các metric, metric hóa được, v.v. (Tôi sẽ không giải thích các khái niệm này ở đây — có thể tìm đọc chúng trong nhiều sách khác nhau — điều quan trọng cần nhớ là các khái niệm này đều rất tự nhiên và có ý nghĩa sâu sắc). Tôi có dạy tô pô (còn gọi là tô pô đại cương, là môn chung cho toàn bộ các SV ngành toán, và chủ yếu nhằm ứng dụng vào giải tích) cho sinh viên toán năm thứ 3 trong một số năm, và những SV nào nắm được cái khái niệm mà tôi vừa liệt kê ra thì coi như học được thành công cái môn tô pô này.

Theo http://zung.zetamu.net




#301654 Để hiểu hơn về phương pháp quy nạp toán học

Gửi bởi Ban Biên Tập trong 29-02-2012 - 23:12

ĐỂ HIỂU HƠN VỀ

PHƯƠNG PHÁP QUY NẠP TOÁN HỌC

Hoàng Ngọc Thế - Đông Anh - Hà Nội


Phương pháp quy nạp toán học là một phương pháp hay và rất hữu dụng. Tuy nhiên, đối với học sinh khối 11 thì đây là nội dung khó hiểu và khó áp dụng. Bài viết này của tôi sẽ giúp các bạn một hướng để hiểu hơn về phương pháp này.

Hình đã gửi

1. Tại sao phải dùng phương pháp quy nạp toán học
Giả sử có 1mệnh đề chứa biến số tự nhiên. Ta cần chứng minh mệnh đề đó. Tại sao phải dùng phương pháp quy nạp toán học?
Để trả lời câu hỏi này, ta xét các bài toán sau:

Bài toán 1.
Thầy giáo kiểm tra bài cũ lớp 11A4 (có 35 học sinh), thầy gọi theo sổ điểm lần lượt các bạn:
  • Triệu Thị Băng
  • Lê Văn Bách
  • Triệu Thị Điềm
  • Đàm Văn Hanh
  • Dương Thị Hường.
Cả 5 bạn ấy đều học bài. Thầy kết luận: “Cả lớp 11A4 học bài”. Thầy kết luận như vậy có hợp lí không? Nếu không làm thế nào để có kết luận đúng.
Giải
Thầy kết luận như vậy là chưa hợp lí vì có thể các bạn từ số thứ tự 6 đến số thứ tự 35 đều học bài, tức là đa phần cả lớp học bài.
Để thu được kết luận đúng, thầy cần kiểm tra cả lớp (bằng cách kiểm tra 15 phút chẳng hạn).


Bài toán 2.
Người ta kiểm tra trên một quần thể ruồi giấm thấy thế hệ đầu tiên có tính trạng mắt đỏ. Kết luận: “Tất cả ruồi giấm ở mọi thế hệ của quần thể này đều mắt đỏ”.
Kết luận như vậy có đúng không? Nếu không làm thế nào để có kết luận đúng?
Giải
Kết luận như vậy chưa chắc đúng vì chưa kiểm tra xem các thế hệ khác có mắt đỏ không?
Ta không thể làm như bài toán 1 vì số lượng ruồi giấm và các thế hệ của quẩn thể là vô số, việc kiểm tra từng cá thể của từng thế hệ là không thể thực hiện được.

Để thu được kết luận đúng, ta làm như sau
+ Kiểm tra với thế hệ thứ nhất (đời F1);
+ Chứng minh sự di truyền của tính trạng mắt đỏ. Tức là chứng minh rằng nếu đời bố mẹ mắt đỏ thì đời con mắt đỏ. Khi đó, chắc chắn tất cả các cá thể ở mọi thế hệ đều mắt đỏ vì thế hệ trước sẽ di truyền lại cho thế hệ sau.

Bài toán 3.
Với $n \in \mathbb{N}*$, chứng minh rằng
$$1 + 2 + ... + n = \dfrac{n(n+1)}{2}, \ \ \ (*).$$
Phân tích
Rõ ràng ta không thể áp dụng cách làm của bài toán 1 cho bài này vì tập các số tự nhiên là vô hạn. Việc kiểm tra tính đúng đắn của $(*)$ với từng số tự nhiên sẽ mất nhiều thời gian và không thể hoàn thành được.

Ta nhận thấy có nét giống nhau giữa tập các số tự nhiên và quần thể ruồi giấm. Tập số tự nhiên có vô hạn phần tử, quần thể ruồi giấm có vô hạn thế hệ. Ta sẽ áp dụng cách làm của bài toán 2 đối với bài toán này.

Coi mệnh đề $(*)$ là một "tính trạng" của "quần thể" các số tự nhiên. Để chứng minh mọi số tự nhiên đều có "tính trạng $(*)$" ta làm như sau:
+ Kiểm tra "tính trạng $(*)$" với "thế hệ đầu (F1)" $n = 1$
+ Chứng minh sự “di truyền” của $(*)$ Tức là chứng minh rằng nếu số $n = k$ có "tính trạng $(*)$" thì $n = k+1$ cũng có "tính trạng $(*)$".
Phương pháp chứng minh như vậy gọi là phương pháp quy nạp toán học. Bạn cũng có thể hiểu phương pháp quy nạp giống như trò chơi Đôminô của người Nhật.

2. Phương pháp và ví dụ
Để chứng minh 1 mệnh đề $A$ đúng với mọi số nguyên dương bằng phương pháp quy nạp toán học, ta thực hiện 2 bước:
- Bước 1 (bước "khởi tạo"). Kiểm tra tính đúng đăn của $A$ với $n=1$
- Bước 2 (bước "di truyền"). Giả sử mệnh đề $A$ đã đúng đến $n = k \geq 1$, ta chứng minh $A$ cũng đúng với $n=k+1$.

Ta sẽ giải bài toán 3.
Bước 1. Với $n=1$, ta có:
$$VT(*)=1=\dfrac{1(1+1)}{2} = VP(*)$$
Vậy $(*)$ đúng với $n = 1$.
Bước 2. Giả sử $(*)$ đã đúng đến $n = k \geq 1$, tức là:
$$1+2+...+k =\dfrac{k(k+1)}{2}, \ \ \ (a).$$
Ta cần chứng minh rằng $(*)$ cũng đúng với $n=k+1$, tức là phải chứng minh:
$$1+2+...+(k+1) =\dfrac{(k+1)(k+2)}{2}, \ \ \ (b).$$
Thật vậy:
$VT(b) = 1+2+...+(k+1) = 1+2+...+k+(k+1)=VT(a)+(k+1)$
$=VP(a)+(k+1)=\dfrac{k(k+1)}{2}+(k+1)=\dfrac{(k+1)(k+2)}{2}=VP(b)$
Ta có đpcm.

Học sinh lớp 11 thường bị vướng khi chứng minh $(b)$. Các em thường không biết bắt đầu từ đâu. Quan sát lời giải bài toán 3, ta thấy lời giải được tiến hành theo logic sau:
$$VT(b)\overset{(1)}{\rightarrow}VT(a)\overset{(2)}{\rightarrow}VP(a)\overset{(3)}{\rightarrow}VP(b)$$

Hình đã gửi

Dấu mũi tên $(1)$, ta sử dụng giả thiết hoặc những phép toán, định nghĩa cơ bản đã học.
Dấu mũi tên $(2)$, ta sử dụng giả thiết quy nạp, tức là dùng $(a)$
Dấu mũi tên $(3)$, ta thường phải biến đổi, ước lượng.

Xin đưa ra thêm một số ví dụ.
Ví dụ 1. Với mọi $n \in \mathbb{N}^*$ ta có: $2^n> n,\ \ \ \ (1)$.
Giải
Bước 1. Với $n = 1$, ta có: $VT = 2, VP = 1$, Vậy $(1)$ đúng.
Bước 2. Giả sử $(1)$ đúng với $n = k \geq 1$, tức là:
$$2^k > k, \ \ \ \ (1a)$$
Ta chứng minh rằng $(1)$ cũng đúng với $n = k+1$. Tức là phải chứng minh:
$$2^{k+1} > k + 1, \ \ \ \ (1b)$$.
Thật vậy, ta có:
$VT(1b) = 2^{k+1} = 2.2^k = 2VT(1a) > 2VP(1a) = 2k > k + 1 = VP(1b)$ (đpcm)
Vậy $(1)$ đúng với mọi $n$ nguyên dương.

Ví dụ 2. Cho dãy số $(u_n)$ xác định bởi:
$$\left\{ \begin{array}{l}
u_1 = \frac{1}{3} \\
u_{n + 1} = \frac{{(n + 1)u_n }}{{3n}},\forall n \ge 1 \\
\end{array} \right.$$
Chứng minh rằng:

$$u_n = \frac{n}{{3^n }},\forall n \ge 1, \ \ \ \ (2)$$
Giải
* Với $n=1$ ta có $VT(2) = u_1 = \frac{1}{3}; VP(2) = \frac{1}{3}$. Vậy $(2)$ đúng với $n=1$.

* Giả sử $(2)$ đúng với $n = k \geq 1$, tức là:
$$u_k = \frac{k}{3^k}, \ \ \ \ (2a)$$
Ta chứng minh rằng $(2)$ cũng đúng với $n = k+1$. Tức là phải chứng minh:
$$u_{k+1} = \frac{k+1}{3^{k+1}}, \ \ \ \ (2b)$$
Thật vậy, ta có:
$u_{k+1} = \frac{{(k + 1)u_k }}{{3k}} = \frac{{(k + 1)k }}{{3k.3^k}} = \frac{k+1}{3^{k+1}}=VP(2b)$ (đpcm)
Vậy $(2)$ đúng với mọi $n$ nguyên dương.

3. Bài tập
Mời các bạn cùng làm thêm các bài tập dưới đây:
Bài tập 1. Chứng minh BĐT Bernoulli:
$$(1+a)^n \geq 1+ na, \forall n \in \mathbb{N}^*$$
Bài tập 2. Chứng minh rằng:
$$\left (11^{n+1}+12^{2n-1} \right )\vdots 133, \forall n \in \mathbb{N}^*$$


#301505 Chuyên đề Cauchy ngược dấu

Gửi bởi Ban Biên Tập trong 28-02-2012 - 22:01

III. MỘT SỐ BÀI TOÁN VÀ LỜI GIẢI HAY
Bài toán 1: Cho $x,y,z$ là 3 số thực dương. Tìm GTLN của biểu thức:
\[P = \frac{{\sqrt {yz} }}{{x + 2\sqrt {yz} }} + \frac{{\sqrt {xz} }}{{y + 2\sqrt {xz} }} + \frac{{\sqrt {xy} }}{{z + 2\sqrt {xy} }}\]

(Đề dự bị khối B – 2010)

LỜI GIẢI
Đây là lời giải của HÀ QUỐC ĐẠT trên Diễn đàn toán học VMF

Ta có: \[\frac{{\sqrt {yz} }}{{x + 2\sqrt {yz} }} = 1 - \frac{x}{{x + 2\sqrt {yz} }} \le 1 - \frac{x}{{x + y + z}}\,\,\,(1)\]
Chứng minh tương tự: \[\frac{{\sqrt {xz} }}{{y + 2\sqrt {xz} }} = 1 - \frac{y}{{y + 2\sqrt {xz} }} \le 1 - \frac{y}{{x + y + z}}\,\,(2);\,\,\frac{{\sqrt {xy} }}{{z + 2\sqrt {xy} }} = 1 - \frac{z}{{z + 2\sqrt {xy} }} \le 1 - \frac{z}{{x + y + z}}\,\,(3)\]
Cộng theo vế $(1)(2)(3)$ ta có: \[2P \le 3 - \frac{{x + y + z}}{{x + y + z}} = 2\,\, \Rightarrow P \le 1\]
Vậy $MinP=1$ khi và chỉ khi $ x=y=z $.

Bài toán 2: Cho 3 số $a,b,c$ là 3 số thực dương. Chứng minh rằng:
$$\frac{{a^3 }}{{a^2 + ab + b^2 }} + \frac{{b^3 }}{{b^2 + ab + c^2 }} + \frac{{c^3 }}{{c^2 + ac + a^2 }} \ge \frac{{a + b + c}}{3}$$
LỜI GIẢI 1

Đây là lời giải của Ispectorgadget trên Diễn đàn toán học VMF

Sử dụng biến đổi và bất đẳng thức $ AM-GM $ cho 3 số ta có:
\[\frac{{{a^3}}}{{{a^2} + ab + {b^2}}} = a - \frac{{{a^2}b + a{b^2}}}{{{a^2} + ab + {b^2}}} \ge a - \frac{{ab(a + b)}}{{3ab}} = a - \frac{{a + b}}{3}\,\,(1)\]
Chứng minh tương tự ta có:
\[\frac{{{b^3}}}{{{b^2} + cb + {c^2}}} \ge b - \frac{{b + c}}{3}\,\,\,\,(2);\frac{{{c^3}}}{{{c^2} + ac + {a^2}}} \ge c - \frac{{a + c}}{3}\,\,\,\,(3)\]
Cộng $(1),(2),(3)$ ta có: $$\frac{a^3}{a^2+ab+b^2}+\frac{b^2}{b^2+bc+c^2}+\frac{c^3}{c^2+ac+a^2}\geq a+b+c-\frac{a+c}{3}-\frac{a+b}{3}-\frac{b+c}{3}=\frac{a+b+c}{3}$$
Ta có được điều phải chứng minh. Đẳng thức xảy ra khi $a=b=c$.

Ta cũng có một lời giải khác khá hay, hoàn toàn không liên quan đến LỜI GIẢI 1 và cũng thu được đánh giá tương tự

LỜI GIẢI 2

Đây là lời giải của vietfrog trên Diễn đàn toán học VMF

Lời giải này sử dụng ''Phương pháp tiếp tuyến''
Giả sử: \[{\frac{{{a^3}}}{{{a^2} + ab + {b^2}}} \ge \frac{{2a - b}}{3}}\]
\[{ \Leftrightarrow {a^3} + {b^3} - {a^2}b - a{b^2} \ge 0}\]
\[{ \Leftrightarrow \left( {a + b} \right){{\left( {a - b} \right)}^2} \ge 0\,\,(*)}\]
Do $ a,b>0 $ nên bất đẳng thức $ (*) $ hiển nhiên đúng. Như vậy giả sử đúng.
Xây dựng 2 bất đẳng thức tương tự với $ b,c $ rồi cộng theo vế ta được:
$$\frac{{{a^3}}}{{{a^2} + ab + {b^2}}} + \frac{{{b^3}}}{{{b^2} + ab + {c^2}}} + \frac{{{c^3}}}{{{c^2} + ac + {a^2}}} \ge \frac{{2a - b}}{3} + \frac{{2b - c}}{3} + \frac{{2c - a}}{3} = \frac{{a + b + c}}{3}$$
Đẳng thức xảy ra khi xảy ra khi $a=b=c$.

Bài toán 3: Cho $a,b,c$ thực dương thỏa mãn $a+b+c=3$. Chứng minh rằng: $$V=\dfrac{a+b+c}{a^2+abc}+\dfrac{a+b+c}{b^2+abc}+\dfrac{a+b+c}{c^2+abc}\geq \dfrac{9}{2}$$

(Đề thi thử ĐH lần 2 trường chuyên Nguyễn Huệ 2007-2008)

LỜI GIẢI

Đây là lời giải của Ispectorgadget trên Diễn đàn toán học VMF
Từ giả thiết ta có:
$$a+b+c=3\geq 3\sqrt[3]{abc}\Leftrightarrow 1\geq abc$$
Do đó: $$V \ge \left( {a + b + c} \right)\left( {\frac{1}{{{a^2} + 1}} + \frac{1}{{{b^2} + 1}} + \frac{1}{{{c^2} + 1}}} \right) = 3\left( {1 - \frac{{{a^2}}}{{{a^2} + 1}} + 1 - \frac{{{b^2}}}{{{b^2} + 1}} + 1 - \frac{{{c^2}}}{{{c^2} + 1}}} \right)$$
$$ \ge 3\left( {1 - \frac{{{a^2}}}{{2a}} + 1 - \frac{{{b^2}}}{{2b}} + 1 - \frac{{{c^2}}}{{2c}}} \right) = 3\left[ {3 - \left( {\frac{{a + b + c}}{2}} \right)} \right] = 9 - \frac{9}{2} = \frac{9}{2}$$
Ta có được điều phải chứng minh. Đẳng thức xảy ra khi $ a=b=c=1 $

Bài toán 4: Cho $a,b$ là 2 số thực dương thay đổi thỏa mãn $a+b=2$. Tìm giá trị nhỏ nhất của: $$M=\dfrac{a^2}{a+1}+\dfrac{b^2}{b+1}$$
LỜI GIẢI 1
Ta có: $$\frac{{{a^2}}}{{a + 1}} = a - \frac{a}{{a + 1}} \ge a - \frac{a}{{2\sqrt a }} = a - \frac{{\sqrt a }}{2}$$
Tương tự với $ b $ ta có được: $$b + 1 \ge b - \frac{{\sqrt b }}{2}$$
Cộng theo vế 2 bất đẳng thức trên ta được $$\dfrac{a^2}{a+1}+\dfrac{b^2}{b+1}=a-\dfrac{\sqrt{a}}{2}+b-\dfrac{\sqrt{b}}{2}=a+b-\dfrac{1}{2}(\sqrt{a}+\sqrt{b})$$
Theo Bất đẳng thức Cauchy-Schwarz ta có:
$$\sqrt{a+b}\geq \dfrac{(\sqrt{a}+\sqrt{b})}{\sqrt{2}}\Leftrightarrow \sqrt{2(a+b)}\geq \sqrt{a}+\sqrt{b}$$
Do đó: $$M\geq (a+b)-\dfrac{\sqrt{2(a+b)}}{2}=2-1=1$$
Như vậy : $MinM = 1$ . Dấu "=" xảy ra khi $a=b=c=1$

Thực ra thì bài toán vẫn được giải quyết bằng bất đẳng thức Cauchy-Schwarz dạng Engel hay vẫn gọi là bất đẳng thức Schwarz:

Với $ x,y,m,n $ là các số dương ta luôn có: \[\frac{{{x^2}}}{m} + \frac{{{y^2}}}{n} \ge \frac{{{{\left( {x + y} \right)}^2}}}{{m + n}}(*)\]
LỜI GIẢI 2

Áp dụng bất đẳng thức Cauchy-Shwarz dạng Engel ta có:\[\frac{{{a^2}}}{{a + 1}} + \frac{{{b^2}}}{{b + 1}} \ge \frac{{{{\left( {a + b} \right)}^2}}}{{a + b + 2}} = \frac{{{2^2}}}{{2 + 2}} = 1\]
Ta cũng có được $MinM=1$ khi và chỉ khi $a=b=1$

Tuy nhiên, khi làm bài thi Đại học thì ta vẫn phải chứng minh lại bất đẳng thức $(*)$. Vì vậy cách chứng minh bằng kỹ thuật Cauchy ngược dấu như trên cũng có thể là một lựa chọn hay.

Bài toán 5: Cho $a,b,c$ là 3 số thực dương; $a,b,c<2$. Chứng minh rằng: $$\dfrac{1}{2-a}+\dfrac{1}{2-b}+\dfrac{1}{2-c}\geq \dfrac{a^2+b^2+c^2}{2}+\dfrac{3}{2}$$
LỜI GIẢI

Ta có: $$\dfrac{1}{b^2+1}=1-\dfrac{b^2}{b^2+1}\geq 1-\dfrac{b^2}{2b}=1-\dfrac{b}{2}=\dfrac{2-b}{2}\Rightarrow \dfrac{1}{2-b}\geq \dfrac{b^2+1}{2}(1)$$
Chứng minh tương tự:
$\dfrac{1}{2-c}\geq \dfrac{c^2+1}{2}$ (2) ;$\dfrac{1}{2-a}\geq \dfrac{a^2+1}{2}$ (3)
Cộng theo vế $(1),(2),(3)$ ta có: $$\dfrac{1}{2-a}+\dfrac{1}{2-b}+\dfrac{1}{2-c}\geq \dfrac{a^2+1+b^2+1+1+c^2}{2}= \dfrac{a^2+b^2+c^2}{2}+\dfrac{3}{2}$$
Ta có điều phải chứng minh. Đẳng thức xảy ra khi các biến bằng 1.

Bài toán 6: Cho $ x,y,z $ là 3 số thực dương thỏa mãn: $ xyz=1 $. Chứng minh bất đẳng thức:
\[\frac{{{x^4}y}}{{{x^2} + 1}} + \frac{{{y^4}z}}{{{y^2} + 1}} + \frac{{{z^4}x}}{{{z^2} + 1}} \ge \frac{3}{2}\]
LỜI GIẢI

Đây là lời giải của phuc_90 trên Diễn đàn toán học VMF

Theo bất đẳng thức $ AM-GM $cho 3 số kết hợp với giả thiết $ xyz=1 $ ta có:
$${x^2}y + {x^2}y + {z^2}x \ge 3\sqrt[3]{{{x^3}.{{\left( {xyz} \right)}^2}}} = 3x$$
Tương tự ta có được:
$$y^2z+y^2z+x^2y \geq 3y$ ; $z^2x+z^2x+y^2z \geq 3z$$
Cộng 3 bất đẳng thức theo vế theo vế ta có:
$$x^2y+y^2z+z^2x \geq x+y+z (*)$$
Theo bất đẳng thức AM-GM ta có: $$\dfrac{x^4y}{x^2+1} = x^2y\left(1-\dfrac{1}{1+x^2}\right) \geq x^2y-\dfrac{1}{2}xy$$
Từ đó suy ra:
$$\sum \dfrac{x^4y}{x^2+1}\geq\sum x^2y-\frac{1}{2}\sum xy\mathop\geq\limits^{(*)}\frac{1}{2}\sum\left(x^2y+y\right)-\frac{1}{2}\sum xy\mathop\geq\limits^{AM-GM}\frac{\sum xy}{2}\mathop\geq\limits^{AM-GM}\frac{3}{2}$$
Đây chính là điều phải chứng minh. Đẳng thức xảy ra khi $ x=y=z=1 $.

Bài toán 7: Cho $a,b,c$ là 3 số thực dương thay đổi thỏa mãn $a+b+c=3$. Chứng minh rằng: $$\frac{a}{{{b^3} + 16}} + \frac{b}{{{c^3} + 16}} + \frac{c}{{{a^3} + 16}} \ge \frac{1}{6}$$
LỜI GIẢI

Đây là lời giải của HÀ QUỐC ĐẠT trên Diễn đàn toán học VMF
Ta có:
$$\dfrac{a}{b^{3}+16}=\dfrac{1}{16}(a-\dfrac{ab^{3}}{b^{3}+16})=\dfrac{1}{16}(a-\dfrac{ab^{3}}{b^{3}+2^{3}+2^{3}})\geq \dfrac{1}{16}(a-\dfrac{ab^{3}}{12b})=\dfrac{1}{16}(a-\dfrac{ab^{2}}{12})$$
Bất đẳng thức cần chứng minh tương đương với
$$\dfrac{1}{16}(3-\dfrac{ab^{2}+bc^{2}+ca^{2}}{12})\geq \dfrac{1}{6}$$
$$\Leftrightarrow ab^{2}+bc^{2}+ca^{2}\leq 4$$
Chứng minh BĐT mạnh hơn:$$ab^{2}+bc^{2}+ca^{2}+abc\leq 4$$
Giả sử $b$ nằm giữa $a$ và $c$.
Ta có: $$a(b-c)(b-a)\leq 0$$
Theo bất đẳng thức $ AM-GM $ $$ab^{2}+bc^{2}+ca^{2}+abc=b(a+c)^{2}+a(b-a)(b-c)\leq b(a+c)^{2}\leq 4$$
Từ đó ta có được điều phải chứng minh.

Bài toán 8: Cho $a,b,c$ là 3 số không nhỏ hơn 1. Chứng minh bất đẳng thức:
\[a(b + c) + b(a + c) + c(a + b) + 2(\frac{1}{{1 + {a^2}}} + \frac{1}{{1 + {b^2}}} + \frac{1}{{{c^2} + 1}}) \ge 9\]
LỜI GIẢI

Ta có: \[\frac{1}{{1 + {a^2}}} + \frac{1}{{1 + {b^2}}} + \frac{1}{{{c^2} + 1}} = 1 - \frac{{{a^2}}}{{1 + {a^2}}} + 1 - \frac{{{b^2}}}{{1 + {b^2}}}1 - \frac{{{c^2}}}{{{c^2} + 1}} = 3 - \left( {\frac{{{a^2}}}{{1 + {a^2}}} + \frac{{{b^2}}}{{1 + {b^2}}} + \frac{{{c^2}}}{{{c^2} + 1}}} \right)\]
Theo bất đẳng thức $ AM-GM $ ta có:\[\frac{{{a^2}}}{{1 + {a^2}}} + \frac{{{b^2}}}{{1 + {b^2}}} + \frac{{{c^2}}}{{{c^2} + 1}} \le \frac{{{a^2}}}{{2a}} + \frac{{{b^2}}}{{2b}} + \frac{{{c^2}}}{{2c}} = \frac{{a + b + c}}{2}\]
Suy ra:\[\frac{1}{{1 + {a^2}}} + \frac{1}{{1 + {b^2}}} + \frac{1}{{{c^2} + 1}} \ge 3 - \frac{{a + b + c}}{2}\]
Ta cần chứng minh:\[a(b + c) + b(a + c) + c(a + b) + 2\left( {3 - \frac{{a + b + c}}{2}} \right) \ge 9\]
\[ \Leftrightarrow a\left( {b + c - 1} \right) + b\left( {a + c - 1} \right) + c\left( {b + a - 1} \right) \ge 3\,\,\,(*)\]
Thật vậy, do $a,b,c \ge 1 \Rightarrow \left( {b + c - 1} \right) \ge 1 \Rightarrow a\left( {b + c - 1} \right) \ge 1$.

Từ đó nhận thấy bất đẳng thức $ (*) $ đúng và suy được bất đẳng thức cần chứng minh.
Đẳng thức xảy ra khi $ a=b=c=1 $

Bài toán 9: Cho $ a,b,c $ là các số dương thỏa mãn ${a^n} + {b^n} + {c^n} = k$ và $n,k \in {N^*}$.
Tìm Min của biểu thức:
\[S = \frac{{{a^n}}}{{1 + n{b^{n + 1}}}} + \frac{{{b^n}}}{{1 + n{c^{n + 1}}}} + \frac{{{c^n}}}{{1 + n{a^{n + 1}}}}\]
LỜI GIẢI

Đây là lời giải của vietfrog trên Diễn đàn toán học VMF
Ta có:
\[S = \sum\limits_{cyc}^{a,b,c} {\frac{{{a^n}}}{{1 + n{b^{n + 1}}}}} = \sum\limits_{cyc}^{a,b,c} {\left( {{a^n} - \frac{{n.{a^n}{b^{n + 1}}}}{{1 + n{b^{n + 1}}}}} \right)} = \sum\limits_{sym}^{a,b,c} {{a^n}} - \sum\limits_{cyc}^{a,b,c} {\frac{{n.{a^n}{b^{n + 1}}}}{{1 + n{b^{n + 1}}}}} \]
\[ \ge k - \sum\limits_{cyc}^{a,b,c} {\frac{{n.{a^n}{b^{n + 1}}}}{{(n + 1)b}}} = k - \frac{n}{{n + 1}}.\sum\limits_{cyc}^{a,b,c} {{a^n}{b^n}} \]
Mặt khác ta có:\[\sum\limits_{cyc}^{a,b,c} {{a^n}{b^n}} = {a^n}{b^n} + {b^n}{c^n} + {a^n}{c^n} \le \frac{{{{\left( {{a^n} + {b^n} + {b^n}} \right)}^2}}}{3} = \frac{{{k^2}}}{3}\]
Suy ra:\[S \ge k - \frac{n}{{n + 1}}.\frac{{{k^2}}}{3}\]
Như vậy: $MinS = k - \frac{n}{{n + 1}}.\frac{{{k^2}}}{3}$ khi và chỉ khi $a = b = c = \frac{k}{3}$.

Bài toán trên có thể tổng quát cho $n$ số dương.

Bài toán 10: Cho $ a_i > 0;i = \overline {1.n} $. Chứng minh rằng: $$\dfrac{{a_1^n }}{{a_1^{n - 1} + (n - 2)a_2^{n - 1} }} + ... + \dfrac{{a_n^n }}{{a_n^{n - 1} + (n - 2)a_1^{n - 1} }} \ge \dfrac{{a_1 + a_2 + ... + a_n }}{{n - 1}}$$
LỜI GIẢI

Đây là lời giải của anh qua trên Diễn đàn toán học VMF

Ta có: $$\dfrac{a_1^n}{a_1^{n-1}+(n-2)a_2^{n-1}}=a_1-\dfrac{(n-2)a_1.a_2^{n-1}}{a_1^{n-1}+(n-2)a_2^{n-1}}$$.
Mà theo bất đẳng thức $ AM-GM $ :
$$a_1^{n-1}+(n-2)a_2^{n-1} \geq (n-1).a_1.a_2^{n-2} $$
Do đó: $$\dfrac{a_1^n}{a_1^{n-1}+(n-2)a_2^{n-1}} \geq a_1-\dfrac{(n-2)a_2}{n-1}$$
Xây đựng các bất đẳng thức tương tự rồi cộng vế với vế ta có điều phải chứng minh.

Trên đây là 10 Bài toán về bất đẳng thức và cực trị có sử dụng kĩ thuật Cauchy ngược dấu. Có thể đó không phải là những lời giải hay nhất, ngắn gọn nhất nhưng mong rằng qua đó, các bạn có thể có thêm những kinh nghiệm mới, cách nhìn nhận mới để từ đó sáng tạo ra những lời giải tuyệt vời. Chúc các bạn thành công.

III. BÀI TẬP ĐỀ NGHỊ

Dưới đây là một số bài tập đề nghị có thể giải bằng cách sử dụng kỹ thuật Cauchy ngược dấu}. Những bài tập này có thể giải quyết bằng nhiều cách nhưng hay thử đặt bút và làm bằng kỹ thuật được nói tới trong bài. Sẽ rất thú vị!

Bài tập 1: Cho $x,y,z$ là 3 số thực dương thay đổi thỏa mãn $x + y+ z=3$. Tìm GTNN của biểu thức:
$$V=\dfrac{x^2}{x+y^2}+\dfrac{y^2}{y+z^2}+\dfrac{z^2}{z+x^2}$$
Bài tập 2: Cho $a,b,c$ là các số thực dương. Chứng minh rằng:$$ \dfrac{a^3}{a^2+b^2}+\dfrac{b^2}{b^2+c^2}+\dfrac{c^3}{c^2+a^2}\geq \dfrac{a+b+c}{2}$$
Bài tập 3: Cho $a,b$ là 2 số thực dương thỏa mãn $ab = 1$.Chứng minh rằng:
$$\dfrac{a^3}{1+b^2}+\dfrac{b^3}{1+a^2}\geq 1$$
Bài tập 4: Cho $ a,b,c $ là các số dương thỏa mãn $ab+bc+ca=3$. Tìm giá trị nhỏ nhất của biểu thức:
\[P=\frac{a}{{1 + 2{b^3}}} + \frac{b}{{1 + 2{c^3}}} + \frac{c}{{1 + 2{a^3}}} \]
Bài tập 5: Cho $a,b,c$ là 3 số thực dương thỏa $a,b,c < 4$. Chứng minh bất đẳng thức: $$\dfrac{1}{4-a}+\dfrac{1}{4-b}+\dfrac{1}{4-c}\geq \dfrac{3}{4}+\dfrac{a^2+b^2+c^2}{16}$$
Bài tập 6: Cho $ a,b,c $ là các số thực dương thỏa mãn: $a+b+c=k$ và $n \in {N^*}$
Tìm giá trị nhỏ nhất của biểu thức sau: \[S = \sum\limits_{cyc}^{a,b,c} {\dfrac{{{a^2}}}{{a + n{b^{n + 1}}}}} \]

Tài liệu tham khảo

1. Cuốn sách ''Sáng tạo Bất đẳng thức'' -- Phạm Kim Hùng
2. Diễn đàn toán học VMF : diendantoanhoc.net
3. Một số đề thi.

Biên tập: Vũ Đình Việt và Trần Trung Kiên
Soạn thảo $\LaTeX$: Vũ Đình Việt.

-----------------------HẾT-----------------------

Mời bạn cùng thảo luận về vấn đề này hoặc đặt câu hỏi cho các tác giả V.Đ. Việt - Tr. Tr. Kiên tại http://diendantoanho...showtopic=67736


#301312 Từ văn học dân gian đến toán học hiện đại

Gửi bởi Ban Biên Tập trong 27-02-2012 - 19:12

Có nhiều vấn đề lớn của toán học hiện đại thực ra đã xuất hiện trong những chuyện dân gian. Chẳng hạn, câu chuyện vui anh chồng tham ăn được bà vợ dùng sợi dây điều khiển, mà hầu như người Việt Nam nào cũng đã từng ít nhất một lần nghe kể, nếu phân tích kĩ sẽ thấy là một bài giảng nhập môn tuyệt vời về Lí thuyết thông tin.

Xưa, một bà vợ có anh chồng rất tham ăn. Tính tham ăn của anh chồng khiến chị vợ nhiều phen xấu hổ. Chị bèn nghĩ ra một kế. Nhân ngày Tết về bên ngoại ăn cỗ, chị ngồi dưới bếp, buộc một sợi dây vào tay chồng và dặn rằng, khi nào chị giật dây một cái thì mới được gắp một miếng. Hôm đó, mọi người ngạc nhiên vì thấy anh chồng ăn uống rất từ tốn. Nào ngờ, chỉ được chừng nửa bữa thì có một chú gà trống chạy qua, mắc chân vào dây. Anh chồng tham ăn được thể gắp lia lịa(theo nhịp dãy chân của chú gà) ! Mẹo hay của chị vợ thế là bị hỏng.

Vấn đề của Lí thuyết thông tin đặt ra trong câu chuyện này là: làm thế nào để mưu kế của chị vợ thành công ngay cả khi không may có chú gà mắc vào dây? Đó chính là một trong những bài toán khó nhất của toán học hiện đại.

Ta thử hình dung một hệ thống điển hình của Lí thuyết thông tin: trước hết, ta có một trung tâm điều khiển, trong trường hợp này là chị vợ. Sau đó là một trung tâm nhận thông tin, chính là chàng tham ăn. Thông tin được truyền qua một kênh truyền tin, chính là sợi dây. Các thông tin được truyền qua kênh truyền tin bằng các tín hiệu, trong trường hợp này là giật dây. Thông tin luôn được truyền dưới dạng mã hoá, ở đây chị vợ đã mã hoá thông tin như sau: giật một cái- gắp một miếng.

Nhưng, một kênh truyền tin, dù hiện đại đến đâu, cũng không thể tuyệt đối chính xác: trung tâm thu nhận thông tin không bao giờ nhận được hoàn toàn chính xác thông tin mà trung tâm điều khiển truyền đi, mà thường bị một nhiễu nào đó. Cái nhiễu mà kênh truyền tin của chị vợ mắc phải chính là con gà tai hại! Vấn đề đặt ra cho chị vợ, cũng như cho Lí thuyết thông tin là: làm thế nào để ngay cả khi bị nhiễu, ta vẫn không đi đến kết quả quá tồi tệ? Nói một cách “hàn lâm” là: làm thế nào để tăng độ tin cậy của kênh truyền tin?

Nếu như quy định của chị vợ không phải là “giật một cái – gắp một miếng” mà là “giật 20 cái – gắp một miếng” thì dù có cái nhiễu là con gà, anh chồng chắc cũng không đến nỗi mang tiếng quá tham ăn! Làm như thế, trong Lí thuyết thông tin gọi là tăng độ thừa để bảo đảm độ tin cậy. Độ thừa ở đây là: lẽ ra chỉ cần giật dây một lần là đủ truyền lệnh gắp một miếng, thì ta phải giật những 20 lần! Nếu chị vợ quá cẩn thận đến mức quy định: giật 100 lần mới gắp một miếng, thì chắc anh chồng được tiếng rất lịch sự, nhưng cũng sẽ mang bụng đói về nhà. Vấn đề nan giải của Lí thuyết thông tin chính là ở chỗ đó: nếu tăng độ thừa để đảm bảo độ tin cậy, thì sẽ bị ảnh hưởng đến tốc độ truyền tin. Trong thực tế, một thông tin chính xác nhưng đến quá muộn có thể là một thông tin vô ích. Vậy, chị vợ nên quy định giật bao nhiêu lần thì anh chồng được gắp một miếng, để sao cho anh ta vừa no bụng, lại vừa được tiếng lịch sự, hay ít nhất là không mang tiếng quá tham ăn, ngay cả khi bị chú gà làm nhiễu kênh truyền tin? Đó chính là bài toán điển hình không chỉ của Lí thuyết thông tin, mà của hầu hết các ngành của Toán học hiện đại: nếu xem mỗi yêu cầu lập thành một miền nào đó, thì phải tìm ra đường biên giới phân chia các miền, sao cho mọi yêu cầu đều được thoả mãn trong một chừng mực chấp nhận được.( Bài toán này chắc không chỉ khó trong toán học, mà cả trong cuộc đời: không thể hy vọng đạt được một cách cao nhất mọi mục tiêu, mà vấn đề là phải làm sao cho hài hoà các mục tiêu đó!).

Để giải bài toán đặt ra, trong những năm gần đây đã xuất hiện nhiều kết quả khá thú vị. Một trong những phương pháp mới là dùng các mã hình học đại số vào Lí thuyết thông tin. Phương pháp này thực sự bất ngờ vì xưa nay, hình học đại số là ngành trừu tượng nhất trong toán học, và ít ai nghĩ lại có thể dùng nó vào một vấn đề rất thực tiễn. Việc dùng hình học đại số để tìm ra biên giới thích hợp trong Lí thuyết thông tin đã góp phần xoá đi biên giới giữa toán học lí thuyết và toán học ứng dụng.

Còn một điều nữa mà tôi chưa nói đến khi kể về hệ thống truyền tin của bà vợ nói trên , đó là vấn đề bảo mật. Nếu có anh chàng nào đó biết được điều giao hẹn của vợ chồng nhà kia và muốn phá vỡ hạnh phúc của họ, hay ít ra chỉ là để trêu chọc thôi, thì anh ta có thể gây nhiễu bằng cách giật dây thật nhanh, để dù bà vợ có “tăng độ thừa” đến đâu, vẫn không thể dứt bỏ được tiếng xấu tham ăn của chồng mình. Vì thế, trong khi truyền tin, nhất thiết phải đặt ra vấn đề bảo mật. Một lần nữa, toán học hiện đại lại có thể giúp ích cho bà vợ bằng cách cung cấp những phương pháp mã hoá hiện đại. Một lúc nào đó, ta sẽ trở lại chủ đề này.

Kho tàng văn học dân gian vô cùng phong phú. Trên đây chỉ là một trong rất nhiều ví dụ về mối liên hệ giữa văn học dân gian và toán học hiện đại. Các bạn thử tìm thêm ví dụ khác nhé!

Theo Hà Huy Khoái




#300885 Dãy số dùng để làm gì?

Gửi bởi Ban Biên Tập trong 25-02-2012 - 08:48

Dãy số và giới hạn của dãy số là một khái niệm mà hầu hết các sinh viên phải học. Hồi tôi còn đi học, cũng phải làm đủ các thứ bài tập về tính giới hạn. Có điều, tôi không hề nhớ là các sách vở hồi đó được học có câu giải thích nào về việc dãy số để làm gì không, hay chỉ đưa ra các định nghĩa, định lý, rồi các bài tập. Không chỉ dãy số, mà phần lớn các sách về các khái niệm toán học khác mà tôi được đọc thời còn là học sinh sinh viên cũng vậy: rất ít giải thích cho người đọc biết là các khái niệm, định lý đó để làm gì.

Việc học các khái niệm mà không biết chúng thực sự dùng để làm gì có cái nguy hiểm là: hoặc là sẽ thấy chán (tại sao mình lại phải học cái dở hơi này, ngoài trừ việc phải thi cho đỗ ?), hoặc là (đối với những người yêu toán, có thể học toán chỉ vì nó “hóc búa” chứ không cần biết là dùng làm gì) thì có một sự nguy hiểm khác là có thể quá sa đà vào những cái mà cuối cùng thì ít dùng đến, trong khi đó lại không dành thời giờ cho những cái có thể cần hơn. Bản thân tôi đã từng là “bệnh nhân” của “căn bệnh” này. Hồi nhỏ, tôi đọc sách về độ đo Lebesgue (quyển sách của Natanzon ?) rất thích, say sưa với các định lý trong đó về các tính chất của các tập đo được, các hàm đo được, và tưởng nhầm đấy là “đỉnh cao của toán học”, có thể dùng nó để “đo mọi thứ” từ cái nhà trở đi. Phải mất rất lâu sau tôi mới hiểu dần ra là lý thuyết đấy cũng chỉ là một trong các công cụ của toán học, và còn bao nhiêu kiến thức khác cần học, nếu chỉ biết có mỗi lý thuyết độ đo thôi thì cũng chẳng làm được gì.

Một trong những nguyên tắc của toán học là: bất kỳ khái niệm quan trọng nào cũng có lý do của nó; nó không phải từ trên trời rơi xuống, mà là xuất phát từ nhu cầu giải quyết các vấn đề nào đó. Hiểu được những lý do sinh ra khái niệm là bước quan trọng trong việc hiểu bản thân khái niệm.

Vậy dãy số (và tất cả các khái niệm kèm theo, như hội tụ, phân kỳ, giới hạn, v.v.) để làm gì ? Công dụng quan trọng nhất của chúng là: để tính toán gần đúng (với độ chính xác cao) những thứ ta cần tính toán !

Ví dụ, ta muốn tính căn bậc 2 của 3. Tất nhiên, thời nay, bấm vào máy tính sẽ cho ra ngay kết quả:
$$\sqrt 3 = 1.732050808$$
Tất nhiên, con số trên cũng không phải là con số hoàn toàn chính xác của $\sqrt 3$, mà chỉ là con số gần đúng với độ chính xác đến 9 chữ số sau dấu phẩy. Ta có thể bắt máy tính tính chính xác hơn, chẳng hạn đến 1000 chữ số sau dấu phẩy, cũng được. Máy tính không có con số đó sẵn trong bộ nhớ, nhưng nó có một thuật toán để tìm ra các giá trị xấp xỉ của $\sqrt 3$, nếu tính càng lâu thì ra kết quả càng chính xác. Thuật toán ở đây như thế nào ? Cùng một bài toán có thể có nhiều thuật toán khác nhau, nhưng các chương trình máy tính, dù có dùng thuật toán nào, thì cũng thường có phần quay vòng, lặp đi lặp lại (loop) trong đó. Chẳng hạn, để tính $\sqrt 3$, ta có thể làm theo thuật toán sau (tính bằng tay cũng được):

Bắt đầu từ một số nguyên gần với $\sqrt 3$, chẳng hạn $a_0 = 1$. Số này là một xấp xỉ thô thiển của với sai số cao.

Ta muốn thay $a_0$ bằng một số $a_1$ khác, là giá trị xấp xỉ của $\sqrt 3$ với sai số ít hơn. Ta đặt $a_1 = a_0 + b_1$ và giải phương trình theo $b_1$:
$$3 = (a_0 + b_1)^2 = a_0^2 + 2a_0b_1 + b_1^2$$
phương trình này là phương trình bậc hai, và lời giải chính xác cần dùng căn thức. Nhưng ta muốn tính với các phép cộng trừ nhân chia thôi. Bởi vậy ta làm như sau: coi $b_1$ là nhỏ, khi đó $b_1^2$ nhỏ đến mức có thể bỏ qua, và ta thay phương trình trên bằng phương trình mới không có $b_1^2$ trong đó:
$$3 = a_0^2 + 2a_0b_1$$
Nghiệm của nó là: $b_1=\frac{3-a_0^2}{2a_0}$ và
$$a_1 = a_0+b_1=\frac{3+a_0^2}{2a_0}$$

Với $a_0 = 1$, ta được $a_1 = \frac{3+1}{2}=2$. Số 2 là một giá trị gần đúng của với sai số còn rất lớn, nhưng sai số này đã nhỏ hơn đáng kể so với sai số của giá trị gần đúng 1.

Khi có $a_1$ rồi, ta tìm số $a_2$ là giá trị gần đúng khác của $\sqrt 3$, chính xác hơn so với $a_1$. Làm hệt như trên, ta được:
$$a_2 = \frac{3+a_1^2}{2a_1} = \frac{3+2^2}{4} = 1.75$$
Con số này đã không còn cách xa lắm giá trị thật của , vì $1.75^2 = 3.0625$.

Tiếp thêm bước nữa, ta có:
$$a_3 = \frac{3+a_2^2}{2a_2} = \frac{3+1.75^2}{3.5} = 1.73214...$$.
Lần này độ chính xác đã đến 3 chữ số sau dấu phẩy.
Tiếp thêm bước nữa:
$$a_4 = \frac{3+a_3^2}{2a_3} = 1.73205081...$$.
Lần này độ chính xác đã lên tới 7 chữ số sau dấu phẩy !

Các con số trên cho thấy thuật toán trên dùng để tính $\sqrt 3$ tuy đơn giản mà rất hiệu nghiệm: chỉ cần làm 4 vòng (từ $a_0$ đi đến $a_4$) là ta đã được kết quả chính xác đến 7 chữ số sau dấu phẩy. Nếu ta cứ tiếp tục lặp đi lặp lại như trên mãi, thì ta được một dãy số $\left ( a_n \right )_{n \in \mathbb{N}}$ hội tụ rất nhanh đến $\sqrt 3$. Để tính $\sqrt 3$ với độ chính xác bất kỳ nào đó theo yêu cầu, chỉ cần lấy một số tự nhiên $n$ tương ứng, rồi tính lặp đi lặp lại như trên cho đến khi ra số $a_n$. Các máy tính làm hoàn toàn tương tự như vậy.

(Bài tập: chứng minh rằng $a_11$ là căn bậc hai của 3 chính xác đến hơn 1000 chữ số sau dấu phẩy).

(Phương pháp tính toán hội tụ nhanh như trên được biết đến từ thời Newton, và mang tên phương pháp Newton. Các phương pháp hội tụ nhanh về sau mang tên Kolmogorov — Nash — Moser dùng trong các vấn đề phương trình đạo hàm riêng hay cơ học thiên thể là dựa trên phương pháp Newton này)

Theo zung.zetamu.net




#300716 Chuyên đề Cauchy ngược dấu

Gửi bởi Ban Biên Tập trong 23-02-2012 - 22:30

KĨ THUẬT CAUCHY NGƯỢC DẤU

Phạm Kim Hùng - Vũ Đình Việt - Trần Trung Kiên


I. LỜI NÓI ĐẦU

Các bạn thân mến!

Bất đẳng thức (BĐT) là một trong những phần kiến thức đặc biệt quan trọng trong Toán Học nói chung và chương trình THPT nói riêng. BĐT cũng là một phần không thể thiếu trong các kỳ thi Toán, thi Đại Học, Cao Đẳng,...

Nói đến BĐT, chúng ta không thể không nhắc đến một BĐT khá quen thuộc đó là BĐT Cauchy (AM-GM). Việc luyện tập tốt về BĐT, giúp cho các bạn có được những tư duy tốt hơn trong việc học tập và cải thiện tốt tâm lý trong phòng thi (khi gặp một bài BĐT khó) ...

Cuốn "Sáng tạo bất đẳng thức" - tác giả: Phạm Kim Hùng, là một trong những cuốn sách rất hay viết về đề tài này.

Để góp phần giúp cho các bạn nắm vững các kiến thức về BĐT này, vận dụng một số kỹ năng, phương pháp để giải quyết các bài toán liên quan đến BĐT, cũng như việc tìm đọc các tài liệu viết về BĐT được dễ dàng hơn, tôi đã tổng hợp và hệ thống ngắn gọn lại các bài tập tiêu biểu của phương pháp gọi là: "Kỹ thuật Cauchy ngược dấu".

Tài liệu được biên soạn nhân sự kiện http://diendantoanhoc.net kỷ niệm 8 năm hoạt động bởi Vũ Đình ViệtTrần Trung Kiên. Có thể coi đây là phần quà dành cho tất cả các bạn. Bài viết sẽ gồm 4 phần chính:

Phần I - LỜI NÓI ĐẦU

Phần II - KỸ THUẬT CAUCHY NGƯỢC DẤU - PHẠM KIM HÙNG: Phần này sẽ trích nguyên mẫu trong sách ''Sáng tạo bất đẳng thức'' - Phạm Kim Hùng.

Phần III - MỘT SỐ BÀI TOÁN VÀ LỜI GIẢI HAY: Ở Phần này gồm các bài toán và lời giải của các thành viên trên http://diendantoanhoc.net cùng một số bài toán sưu tầm.

Phần IV - BÀI TẬP ĐỀ NGHỊ

Việc biên soạn không thể tránh khỏi những thiếu sót, rất mong những ý kiến đóng góp của các bạn! Mọi đóng góp xin gửi về địa chỉ: [email protected]

Trân trọng cảm ơn!

II. KỸ THUẬT CAUCHY NGƯỢC DẤU - PHẠM KIM HÙNG

Phần II này được trích nguyên mẫu trong cuốn sách ''Sáng tạo Bất đẳng thức'' của tác giả Phạm Kim Hùng.

Chúng ta sẽ xem xét bất đẳng thức $AM - GM$ và một kĩ thuật đặc biệt - kỹ thuật Cauchy ngược dấu. Đây là một trong những kĩ thuật hay, khéo léo, mới mẻ và ấn tượng nhất của bất đẳng thức $AM-GM$. Hãy xem các ví dụ cụ thể sau:

Ví dụ 1: Các số dương $ a,b,c $ thỏa mãn điều kiện $ a+b+c=3 $. Chứng minh bất đẳng thức: \[\frac{a}{{1 + {b^2}}} + \frac{b}{{1 + {c^2}}} + \frac{c}{{1 + {a^2}}} \ge \frac{3}{2}\]
LỜI GIẢI

Ta không thể sử dụng trực tiếp bất đẳng thức $AM-GM$ với mẫu số vì bất đẳng thức sẽ đổi chiều\[\frac{a}{{1 + {b^2}}} + \frac{b}{{1 + {c^2}}} + \frac{c}{{1 + {a^2}}} \le \frac{a}{{2b}} + \frac{b}{{2c}} + \frac{c}{{2a}} \ge \frac{3}{2}?!\]
Tuy nhiên, rất may mắn ta có thể dùng lại bất đẳng thức đó theo cách khác\[\frac{a}{{1 + {b^2}}} = a - \frac{{a{b^2}}}{{1 + {b^2}}} \ge a - \frac{{a{b^2}}}{{2b}} = a - \frac{{ab}}{2}\]
Ta đã sử dụng bất đẳng thức $ AM-GM $cho 2 số $1 + {b^2} \ge 2b$ ở dưới mẫu nhưng lại có được một bất đẳng thức thuận chiều? Sự may mắn ở đây là một cách dùng ngược bất đẳng thức $ AM-GM $, một kĩ thuật rất ấn tượng và bất ngờ. Nếu không sử dụng phương pháp này thì bất đẳng thức trên sẽ rất khó và dài.

Từ bất đẳng thức trên, xây dựng 2 bất đẳng thức đương tự với $ b,c $ rồi cộng cả 3 bất đẳng thức lại suy ra:\[\frac{a}{{1 + {b^2}}} + \frac{b}{{1 + {c^2}}} + \frac{c}{{1 + {a^2}}} = a + b + c - \frac{{ab + bc + ac}}{2} \ge \frac{3}{2}\]
vì ta có ${ab + bc + ac \le 3}$. Đẳng thức xảy ra khi $a=b=c=1$.
Với cách làm trên có thể xây dựng bất đẳng thức tương tự với 4 số.

Ví dụ 2: Các số dương $ a,b,c,d $ thỏa mãn điều kiện $ a+b+c=3 $. Chứng minh bất đẳng thức: \[\frac{a}{{1 + {b^2}}} + \frac{b}{{1 + {c^2}}} + \frac{c}{{1 + {d^2}}} + \frac{d}{{1 + {a^2}}}\ge 2\]
Và nếu không dùng kĩ thuật Cauchy ngược dấu thì gần như bài toán này không thể giải được theo cách thông thường được. Kĩ thuật này thực sự hiệu quả với các bài toán bất đẳng thức hoán vị.

Ví dụ 3: Chứng minh với mọi số thực dương $ a,b,c,d $ thỏa mãn điều kiên $a+b+c+d=4 $ ta có:\[\frac{a}{{1 + {b^2}c}} + \frac{b}{{1 + {c^2}d}} + \frac{c}{{1 + {d^2}a}} + \frac{d}{{1 + {a^2}b}} \ge 2\]
LỜI GIẢI

Theo bất đẳng thức $ AM-GM $
\[\frac{a}{{1 + {b^2}c}} = a - \frac{{a{b^2}c}}{{1 + {b^2}c}} \ge a - \frac{{a{b^2}c}}{{2b\sqrt c }} = a - \frac{{ab\sqrt c }}{2}\]
\[ \ge a - \frac{{b\sqrt {a.ac} }}{2} \ge a - \frac{{b\left( {a + ac} \right)}}{4}\]
\[\, \Rightarrow \frac{a}{{1 + {b^2}c}} \ge a - \frac{1}{4}\left( {ab + abc} \right)\]
Hoàn toàn tương tự ta có thêm 3 bất đẳng thức sau:
\[\frac{b}{{1 + {c^2}d}} \ge b - \frac{1}{4}\left( {bc + bcd} \right),\frac{c}{{1 + {d^2}a}} \ge c - \frac{1}{4}\left( {cd + cda} \right),\frac{d}{{1 + {a^2}b}} \ge d - \frac{1}{4}\left( {da + dab} \right)\]
Cộng vế cả 4 bất đẳng thức trên ta được
\[\frac{a}{{1 + {b^2}c}} + \frac{b}{{1 + {c^2}d}} + \frac{c}{{1 + {d^2}a}} + \frac{d}{{1 + {a^2}b}} \ge a + b + c + d - \frac{1}{4}\left( {ab + bc + cd + da + abc + bcd + cda + dab} \right)\]
Từ bất đẳng thức $ AM-GM $ dễ dàng suy ra các bất đẳng thức:
$${ab + bc + cd + da \le \frac{1}{4}{{\left( {a + b + c + d} \right)}^2} = 4}$$
$${abc + bcd + cda + dab \le \frac{1}{{16}}{{\left( {a + b + c + d} \right)}^3} = 4}$$
Do đó\[\frac{a}{{1 + {b^2}c}} + \frac{b}{{1 + {c^2}d}} + \frac{c}{{1 + {d^2}a}} + \frac{d}{{1 + {a^2}b}} \ge a + b + c + d - 2 = 2\]
Đẳng thức xảy ra khi và chỉ khi $ a=b=c=d=1 $.

Ví dụ 4: Chứng minh với mọi số thực dương $ a,b,c,d $ tac luôn có: \[\frac{{{a^3}}}{{{a^2} + {b^2}}} + \frac{{{b^3}}}{{{b^2} + {c^2}}} + \frac{{{c^3}}}{{{c^2} + {d^2}}} + \frac{{{d^3}}}{{{d^2} + {a^2}}} \ge \frac{{a + b + c + d}}{2}\]
LỜI GIẢI

Sử dụng bất đẳng thức $ AM-GM $ với 2 số
\[\frac{{{a^3}}}{{{a^2} + {b^2}}} = a - \frac{{a{b^2}}}{{{a^2} + {b^2}}} \ge a - \frac{{a{b^2}}}{{2ab}} = a - \frac{b}{2}\]
Xây dựng 3 bất đẳng thức tương tự với $ b,c,d $rồi cộng theo vế các bất đẳng thức lại ta được:\[\frac{{{a^3}}}{{{a^2} + {b^2}}} + \frac{{{b^3}}}{{{b^2} + {c^2}}} + \frac{{{c^3}}}{{{c^2} + {d^2}}} + \frac{{{d^3}}}{{{d^2} + {a^2}}} \ge a - \frac{b}{2} + b - \frac{c}{2} + c - \frac{d}{2} + d - \frac{a}{2} = \frac{{a + b + c + d}}{2}\]
Ta có được điều phải chứng minh. Đẳng thức xảy ra khi tất cả các biến bằng nhau.
Một bất đẳng thức cùng dạng trên là:\[\frac{{{a^4}}}{{{a^3} + 2{b^3}}} + \frac{{{b^4}}}{{{b^3} + 2{c^3}}} + \frac{{{c^4}}}{{{c^3} + 2{d^3}}} + \frac{{{d^4}}}{{{d^3} + 2{a^3}}} \ge \frac{{a + b + c + d}}{3}\]

Ví dụ 5: Cho $ a,b,c \ge 0 $ và $ a+b+c=3 $. Chứng minh:
\[\frac{{{a^2}}}{{a + 2{b^2}}} + \frac{{{b^2}}}{{b + 2{c^2}}} + \frac{{{c^2}}}{{b + 2{a^2}}} \ge 1\]
LỜI GIẢI

Sử dụng biến đổi và bất đẳng thức $ AM-GM $ cho 3 số:
\[\frac{{{a^2}}}{{a + 2{b^2}}} = a - \frac{{2a{b^2}}}{{a + 2{b^2}}} \ge a - \frac{{2a{b^2}}}{{3\sqrt[3]{{a{b^4}}}}} = a - \frac{2}{3}{\left( {ab} \right)^{\frac{2}{3}}}\]
Hoàn toàn tương tự ta cũng có 2 bất đẳng thức:
\[\frac{{{b^2}}}{{b + 2{c^2}}} \ge b - \frac{2}{3}{\left( {bc} \right)^{\frac{2}{3}}},\frac{{{c^2}}}{{c + 2{a^2}}} \ge c - \frac{2}{3}{\left( {ca} \right)^{\frac{2}{3}}}\]
Do đó ta chỉ cần chứng minh:
\[a + b + c - \frac{2}{3}\left( {{{\left( {ab} \right)}^{\frac{2}{3}}} + {{\left( {bc} \right)}^{\frac{2}{3}}} + {{\left( {ca} \right)}^{\frac{2}{3}}}} \right) \ge 1\]
\[ \Leftrightarrow {\left( {ab} \right)^{\frac{2}{3}}} + {\left( {bc} \right)^{\frac{2}{3}}} + {\left( {ca} \right)^{\frac{2}{3}}} \le 3\]
Nhưng bất đẳng thức này hiển nhiên đúng, vì theo bất đẳng thức $ AM-GM $:
\[a + ab + b \ge 3{\left( {ab} \right)^{\frac{2}{3}}},b + bc + c \ge 3{\left( {bc} \right)^{\frac{2}{3}}},c + ca + a \ge 3{\left( {ca} \right)^{\frac{2}{3}}}\]
Ngoài ra dễ thấy $ ab + bc + ca \le 3 $ nên ta có điều phải chứng minh.
Đẳng thức xảy ra khi $ a=b=c=1 $

Kết quả của bài toán vẫn đúng khi thay giả thiết $ a+b+c=3 $ bởi $ ab+bc+ca=3 $ hoặc $ \sqrt a + \sqrt b + \sqrt c = 3 $, trường hợp sau khó hơn một chút. Ta có thêm một bất đẳng thức khác cùng dạng trên.

Ví dụ 6: Cho $ a,b,c \ge 0 $ và $ a+b+c=3 $. Chứng minh rằng:
\[\frac{{{a^2}}}{{a + 2{b^3}}} + \frac{{{b^2}}}{{b + 2{c^3}}} + \frac{{{c^2}}}{{b + 2{a^3}}} \ge 1\]
LỜI GIẢI

Chứng minh tương tự đưa bất đẳng thức về:
\[b\sqrt[3]{{{a^2}}} + c\sqrt[3]{{{a^2}}} + a\sqrt[3]{{{c^2}}} \le 3\]
Sau đó áp dụng bất đẳng thức $ AM-GM $ ta có:
\[b{a^{\frac{2}{3}}} \le b\left( {2a + 1} \right),c{b^{\frac{2}{3}}} \le c\left( {2b + 1} \right),a{c^{\frac{2}{3}}} \le a\left( {2c + 1} \right)\]
Cộng cả ba vế bất đẳng thức trên được điều phải chứng minh.

Ví dụ 7: Chứng minh với mọi số thực dương $ a,b,c $ có tổng bằng 3 thì:
\[\frac{{a + 1}}{{{b^2} + 1}} + \frac{{b + 1}}{{{c^2} + 1}} + \frac{{c + 1}}{{{a^2} + 1}} \ge 3\]
LỜI GIẢI

Theo bất đẳng thức $ AM-GM $ dễ thấy
\[\frac{{a + 1}}{{{b^2} + 1}} = a + 1 - \frac{{\left( {a + 1} \right){b^2}}}{{{b^2} + 1}} \ge a + 1 - \frac{{{b^2}\left( {a + 1} \right)}}{{2b}} = a + 1 - \frac{{ab + b}}{2}\]
Tương tự ta có 2 bất đẳng thức nữa với $ b,c $ rồi cộng lại ta được:
\[\frac{{a + 1}}{{{b^2} + 1}} + \frac{{b + 1}}{{{c^2} + 1}} + \frac{{c + 1}}{{{a^2} + 1}} \ge \left( {a + 1 - \frac{{ab + b}}{2}} \right) + \left( {b + 1 - \frac{{bc + c}}{2}} \right) + \left( {c + 1 - \frac{{ca + a}}{2}} \right)\]
\[ = 3 + \frac{{a + b + c - ab - bc - ca}}{2} \ge 3\]
Đẳng thức xảy ra khi $ a=b=c=1$.

Ví dụ 8: Chứng minh rằng với mọi $ a,b,c,d $ dương có tổng bằng 4 thì:
\[\frac{{a + 1}}{{{b^2} + 1}} + \frac{{b + 1}}{{{c^2} + 1}} + \frac{{c + 1}}{{{d^2} + 1}} + \frac{{d + 1}}{{{a^2} + 1}} \ge 2\]
Cũng bằng phương pháp tương tự ta có bất đẳng thức sau đây.

Ví dụ 9: Chứng minh rằng với mọi $ a,b,c,d $dương có tổng bằng 4 thì:
\[\frac{1}{{{a^2} + 1}} + \frac{1}{{{b^2} + 1}} + \frac{1}{{{c^2} + 1}} + \frac{1}{{{d^2} + 1}} \ge 2\]
LỜI GIẢI

Thật vậy ta có đánh giá sau:
\[\frac{1}{{{a^2} + 1}} = 1 - \frac{{{a^2}}}{{{a^2} + 1}} \ge 1 - \frac{{{a^2}}}{{2a}} = 1 - \frac{a}{2}\]
Sau đó chỉ cần làm tương tự với $ b,c,d $ rồi cộng lại ta được điều phải chứng minh.
Đẳng thức xảy ra khi $ a=b=c=d=1 $.

Kĩ thuật Cauchy ngược dấu là một kĩ thuật giúp giải quyết bài toán theo lối suy nghĩ nhẹ nhàng và trong sáng, các kết quả làm bằng kĩ thuật này nói chung rất khó có thể làm được theo cách khác, hoặc phải làm theo cách khá dài.

Trích ''Sáng tạo Bất đẳng thức--Phạm Kim Hùng''

Còn tiếp ...

Mời bạn cùng thảo luận về vấn đề này hoặc đặt câu hỏi cho các tác giả V.Đ. Việt - Tr. Tr. Kiên tại: http://diendantoanho...showtopic=67736


#299270 Suy nghĩ về vai trò của toán học trong xã hội

Gửi bởi Ban Biên Tập trong 13-02-2012 - 21:17

Bài nói của GS Hà Huy Khoái ở Hội thảo về Phổ thông chuyên Toán, ĐHQG Hà Nội tổ chức, tháng 1/1998.

Câu hỏi “Toán học phổ thông: tồn tại hay không tồn tại?” đặt ra ở một hội nghị bàn về “Giảng dạy toán học phổ thông và toán học phổ thông với toán học hiện đại”, chắc chắn sẽ gây nhiều tranh cãi. Tuy nhiên, người viết bài này hy vọng sẽ tránh được phần nào “búa riù”, bởi lẽ bản báo cáo không những nhằm mục đich “chứng minh” không tồn tại “toán học phổ thông”, mà còn “chứng minh” sự không tồn tại của “toán học hiện đại”. Nói cách khác, chỉ tồn tại một Toán học duy nhất. Chúng tôi cũng mạnh dạn góp một vài ý kiến rất chủ quan của mình về việc làm thế nào bồi dưỡng cho học sinh lòng say mê toán học từ những bài học ở nhà trường phổ thông.

Tồn tại khá phổ biến trong học sinh quan niệm cho rằng, toán học đã là một “lâu đài đẹp đẽ”, khó có thể phát kiến thêm điều gì ở đó, và toán học bao giờ cũng rất xa rời với thực tiễn. Vì thế, để hướng cho các em say mê với toán học, không gì hơn là cho các em thấy rõ, từ những trang sách nhà trường đến những ứng dụng lớn lao nhất của toán học chỉ là một bước nhỏ, và hầu như ai cũng có thể vượt qua bước đó, chỉ cần suy nghĩ sâu hơn một chút! Đó cũng là nội dung chủ yếu mà báo cáo này muón đề cập đến, thông qua việc trình bày một số thành tựu quan trọng nhất của toán học, mà một học sinh với kiến thức phổ thông có thể hiểu rõ, ít nhất là về ý tưởng.

1. Từ Eratosthènes đến mật mã khoá công khai.
Ngay từ bậc tiểu học, chúng ta đã biết, sàng Eratosthenes cho cách tìm tất cả các số nguyên tố. Và bất kì học sinh nào cũng biết phân tích một số nguyên ra thừa số nguyên tố. Bài toán tưởng chừng như quá đơn giản, và không còn gì để nghiên cứu nữa. Nhưng phải chăng, việc chúng ta kết thúc bài giảng tại đó là chưa hợp lí? Trong thời đại mà tin học xâm nhập vào mọi lĩnh vực của đời sống, thiết tưởng nên để cho học sinh biết rằng thời gian để phân tích một số ra thừa số nguyên tố nhiều khi thật khó chấp nhận. Chẳng hạn, thời gian cần thiết để phân tích một số có khoảng 200 chữ số ra thừa số nguyên tố (với một máy tính tốc độ 1 triệu phép tính trên 1 giây) là… 3,8 tỷ năm! Vậy chúng ta đành bó tay trước những số lớn như vậy sao? Ở đây, toán học đã “lợi dụng “ sự yếu kém của máy tính, và đó là nguyên nhân ra đời của một hiện tượng gây nhiều tiếng vang: các hệ mật mã khoá công khai. Nói một cách vắn tắt, tư tưởng của nó là như sau. Để có thể tiếp nhận thông tin mật mà người khác gửi đến cho mình, mỗi người chỉ cần công bố công khai một “khoá lập mã”, là một số nguyên n đủ lớn (khoảng 200 chữ số). Ai cũng có thể mã hoá các thông báo và chuyển cho người cần nhận khi biết khoá n đó. Tuy vậy, để đọc được thông báo đó, cần biết cách phân tích số n ra thừa số nguyên tố, và việc làm này mất hàng tỷ năm! Với người đã công bố khoá thì vấn đề quá đơn giản: số n chính là số mà anh ta nhận được bằng cách nhân hai số nguyên tố đủ lớn đã chọn sẵn. Và như vậy, anh ta chỉ cần giữ bí mật hai số nguyên tố đó, không một ai khác biết các số đó. Điều này thực sự khác với các hệ mật mã cổ điển, khi mà mọi người cùng trong một hệ thống đều nắm được bí mật của nhau, và do đó, bí mật này rất dễ bị lộ.

Sự ra đời của các hệ mật mã khoá công khai là một cuộc cách mạng lớn trong thông tin. Vậy mà để giải thích nó, chỉ cần đến kiến thức của học sinh cấp hai! Điều này đã thực sự xoá nhoà ranh giới giữa toán học “phổ thông” và toán học “hiện đại”, thậm chí, ranh giới giữa toán học lí thuyết và toán học ứng dụng. Một công trình nghiên cứu toán học thuần tuý có thể ngay lập tức bước vào thực tiễn.

Vậy nhưng con đường từ toán học đến thực tiễn không phải bao giờ cũng nhanh chóng và bằng phẳng như vậy. Tôi muốn nói dến một trong những ứng dụng vĩ đại nhất trong lịch sử, và thời gian đi từ lí thuyết đến thực tiễn là vào khoảng 2000 năm! Và một lần nữa, lại là ví dụ cho thấy từ trang sách toán phổ thông có thể đi đến những phát kiến vĩ đại

2. Từ Apollonius đến KeplerNewton.
Các thiết diện côníc đã được nhà toán học cổ Hy Lạp Apollonius nghiên cứu vào thế kỉ thứ 3 trước công nguyên. Trong nhiều thế kỉ, đó là một lí thuyết đẹp, nhưng cũng giống như nhiều lí thuyết toán học khác, chỉ được xem như các “trò chơi của trí tuệ”. Mãi đến đầu thế kỉ 17, lợi ích của lí thuyết này mới được chứng minh, khi Johannes Kepler phát minh ra luật chuyển động của các hành tinh. Thầy học của ông, nhà thiên văn Tycho Brahe đã tiến hành đo đạc trong vòng 20 năm tại đài thiên văn Uraniborg về vị trí các hành tinh trong hệ mặt trời, và đi đến kết luận rằng, các hành tinh chuyển động theo quỹ đạo vòng tròn. Sau khi Tycho Brahe qua đời, Kepler lãnh đạo đài thiên văn và ông không bằng lòng với kết luận cho rằng, độ lệch khỏi vòng tròn của quỹ đạo các hành tinh mà đài quan sát được chỉ là sai số đo đạc. Vốn là người rất say mê lí thuyết các đường côníc và hiểu rõ rằng, các đường ellip với hai tiêu cự rất gần nhau trông rất giống đường tròn, Kepler nghi ngờ rằng, các quỹ đạo đã được xem là đường tròn đó rất có thể lại là các ellip. Sau khi kiểm tra lại kĩ lưỡng, Kepler đi đến phát minh vào loại vĩ đại nhất trong lịch sử: các hành tinh chuyển động theo quỹ đạo ellip. Phát kiến này được Newton chứng minh vào cuối thế kỉ 17 bằng lí thuyết vạn vật hấp dẫn.

Như vậy, bằng trí tuệ của mình, Apollonius đã phát hiện ra những đường cong vĩ đại của vũ trụ, và đẩy nhanh sự phát minh ra một trong những quy luật quan trọng nhất của tự nhiên.

3. Từ Archimede đến Einstein.
Nếu như những ví dụ trên đây cho thấy, đằng sau các khái niệm và kiến thức toán học phổ thông có thể ẩn náu những thành tựu hiện đại nhất của toán học và những phát kiến vĩ đại nhất, thì ví dụ tiếp theo sẽ lại một lần nữa cho học sinh thấy rằng ”lâu đài toán học” không phải đã hoàn hảo như ta tưởng, và ở đó còn nhiều việc cần làm.
Khi bắt đầu với bộ môn hình học, chúng ta đều giảng về một tiên đề rất trực quan, đó là tiên đề Archimede: khi dùng một đoạn thẳng làm đơn vị để đo một đoạn thẳng khác dài hơn, ta sẽ được một số nguyên lần đơn vị đo, và còn lại một đoạn có độ dài bé hơn đơn vị. Chắc ít ai nghi ngờ gì về tiên đề đã nêu. Tuy nhiên, tình hình sẽ thay đổi hẳn khi ta suy nghĩ sâu hơn một chút về sự thống nhất của thế giới vĩ mô và vi mô.

Một trong những bài toán cơ bản mà Einstein có ước mơ giải quyết là xây dựng một lí thuyết trường thống nhất cho cả thế giới vĩ mô và thế giới vi mô. Dĩ nhiên, trong một lí thuyết thống nhất như vậy chúng ta phải dùng “khoảng cách” thống nhất. Điều gì sẽ xẩy ra, nếu khoảng cách này thoả mãn tiên đề Archimede? Khi đo khoảng cách trong thế giới vi mô, ta thường dùng “thang Planck”, bằng khoảng 10-35 cm. Hãy hình dung việc lấy thang đó làm đơn vị để đo khoảng cách giữa các vì sao. Ta sẽ được một số hữu hạn lần đơn vị đo, có thể “còn lại” một khoảng bé hơn 10-35 cm? Lần này, trực giác khó làm cho ta chấp nhận, như đã chấp nhận tiên đề Archimede bằng trực giác. Vậy, phải chăng để xây dựng được lí thuyết trường thống nhất, ta cần một khái niệm khoảng cách mà trong đó tiên đề Archimede không còn đúng nữa? Câu hỏi này đã được nhiều nhà vật lí nghiên cứu, và trong những năm gần đây đã ra đời bộ môn vật lí không Archimede. Khoảng cách được dùng trong đó chính là khoảng cách không thoả mãn tiên đề Archimede (khoảng cách p-adic) đã được xây dựng từ lâu trong toán học. Một điều thú vị là, định lí Ostrovski khẳng định rằng, nếu trên tập hợp các số hữu tỉ, ta cho một khoảng cách thoả mãn các tiên đề thông thường thì đó hoặc phải là khoảng cách thông thường, hoặc là khoảng cách p-adic với một số nguyên tố p nào đó. Như vậy, việc đưa thêm các khoảng cách p-adic đã vét cạn mọi khoảng cách có thể được cho trên tập hợp các số hữu tỷ. Khoảng cách p-adic có ứng dụng không chỉ trong các bài toán hình học, mà còn cả trong số học. Thực ra, khoảng cách này bắt đầu từ những nghiên cứu số học.

Như vậy, ngay đằng sau một tiên đề của hình học phổ thông, ta đã thấy mầm mống của sự xuất hiện một ngành mới của toán học hiện đại, và thậm chí, một ngành vật lí mới.

Có thể dẫn ra nhiều ví dụ tương tự để chứng minh rằng, không có khoảng cách nào giữa toán học phổ thông và toán học hiện đại. Vậy thì, chúng ta cần giảng dạy như thế nào để học sinh phổ thông yêu thích môn toán và có hình dung đúng đắn về toán học hiện đại? Đây là một vấn đề quá lớn, và chúng tôi chỉ xin mạnh dạn nêu vài ý kiến chủ quan, xuất phát từ sự phân tích trên đây về quan hệ giữa toán học phổ thông và toán học hiện đại.

4. Dạy theo Bourbaki hay theo các bà nội trợ?
Đã một thời, những bài tập ở phổ thông thường mô phỏng loại toán của các bà nội trợ: một người đi chợ mang theo 100 đồng, dùng hết số tiền đó và mua được 36 con vừa gà vừa chó. Giá mỗi con chó là 4 đồng, giá mỗi con gà là 2 đồng. Hỏi người đó mua mấy con gà, mấy con chó? Thật là một bài toán xa thực tế, vì chẳng mấy ai mua bán như vậy. Dĩ nhiên, cũng có thể đặt những bài toán có vẻ thực tế hơn, nhưng dù sao, vẫn là “loại toán của các bà nội trợ”. Đó là lí do mà trong những năm gần đây, người ta có xu hướng đưa vào chương trình toán những vấn đề có vẻ gần “thực tiễn” hơn. Xu hướng này đặc biệt phổ biến ở Mỹ. Kết quả của phương pháp giảng dạy này còn phải tranh cãi nhiều, nhưng tưởng cũng cần nhắc lại câu của nhà thơ Maiacôpxki khi nói về sự cách tân trong thơ Nga: “ Người đầu tiên phát minh ra 2+2=4 là một nhà toán học vĩ đại, dù anh ta phát minh ra điều đó nhờ việc cộng 2 điếu thuốc lá với 2 điếu thuốc lá. Còn người sau đó phát hiện ra 2 cái đầu tàu hoả cộng 2 đầu tàu hoả bằng 4 đầu tàu hoả thì đã không còn là nhà toán học nữa!” Như vậy, ngay nhà thơ vĩ đại cũng thấy rằng, điều quan trọng ở đây là cấu trúc chứ không phải bản thân các đối tượng đề cập đến trong bài toán. Những người phản đối phương pháp dạy mới ở Mỹ cho rằng, người ta đang dạy cho học sinh thứ toán học “đầu tàu”, và tưởng nhầm là hay hơn toán học của các bà nội trợ.

Nhưng, cũng tồn tại khá phổ biến quan niệm ngược lại. Sự chú ý đặc biệt đến việc cho học sinh làm quen dần với các cấu trúc đại số đã dẫn đến quan niệm về giảng dạy theo “tinh thần Bourbaki”. Trong vài thập kỉ gần đây, quan niệm này gây sự chú ý rộng rãi trong cộng đồng các nhà nghiên cứu và giảng dạy toán học. Những ngưòi ủng hộ quan niệm đó đã có công rất lớn trong việc rèn luyện cho học sinh tư duy trưù tượng, đặc biệt là tránh một số sai lầm do trực giác gây ra. Tuy nhiên, việc đưa vào chương trình phổ thông những khái niệm trừu tượng theo kiểu tiên đề cũng không tranh khỏi gây nhiều tranh cãi. Thứ nhất, không ít người đã đồng nhất “trừu tượng” và “hiện đại”. Họ cho rằng, những gì hiện đại thì phải trừu tượng, và ngược lại. Thực ra, một vài ví dụ nhỏ trong bài này đã phần nào cho thấy sự phát triển hiện đại của toán học nằm trong nhu cầu nội tại của toán học và trong nhu cầu của thực tiễn, và một thành tựu, một lĩnh vực được xem là hiện đại hay không khi nó đáp ứng đến mức độ nào các nhu cầu đó, chứ tuyệt nhiên không phải ở mức độ trừu tượng của nó. Thực ra, trong nghiên cứu, các nhà toán học chỉ dùng trừu tượng ở mức độ “tối thiểu cần thiết”. Qua việc chỉ ra một số thành tựu hiện đại nhất của toán học mà một học sinh phổ thông có thể hiểu được, chúng ta cũng thấy rằng, có thể làm cho học sinh phổ thông hiểu toán học hiện đại là gì, mà không đòi hỏi phải viện đến các khái niệm trừu tượng. Vả lại, một khi học sinh chưa được trang bị đủ “ hình” cần thiết thì việc tiếp thu các khái niệm trừu tượng thường mang nặng ý nghĩa hình thức. Điều này dễ dần đến việc hiểu sai bản chất của toán học. Nói cho cùng, toán học là sản phẩm của thực tiễn, và nó thực sự dễ hiểu khi ta mô tả nó một cách giản dị và cụ thể.

Tóm lại, mục tiêu của chúng ta là, một mặt, trang bị cho học sinh những kiến thức toán học cần thiết, và những kiến thức đó càng gần với thực tiễn bao nhiều thì càng tốt bấy nhiêu, mặt khác, làm cho học sinh hiểu được bản chất của toán học và say mê học toán. Muốn vậy, không thể chỉ dạy cho học sinh “toán học phổ thông”, bởi lẽ không có một hàng rào nào ngăn cách toán học phổ thông với toán học hiện đại. Chỉ có điều, cần hiểu đúng thế nào là hiện đại, để tránh “trừu tượng hoá” chương trình toán một cách không cần thiết. Đằng sau mỗi bài toán của các bà nội trợ đều ẩn náu một phát minh vĩ đại của toán học hiện đại. Song, đối với người thầy, làm cho học sinh hiểu được điều đó quả là một nhiệm vụ cực kì khó khăn!


#299056 Japan Mathematical Olympiad Finals 2012

Gửi bởi Ban Biên Tập trong 12-02-2012 - 14:27

Kỳ thi Japan Mathematical Olympiad Finals 2012 vừa diễn ra ngày 11/02/2012. Dưới đây là toàn bộ đề thi.

Câu 1:
Cho tam giác $ABC$, các tiếp tuyến tại đỉnh $A$ của đường tròn ngoại tiếp cắt $BC$ tại $P$. Gọi $P, Q$ lần lượt là điểm đối xứng của $P$ qua các $AB, AC$. Chứng minh rằng $BC$ vuông góc với $QR$.

Câu 2:
Tìm tất cả các hàm số: $ f : \mathbb{R} \mapsto \mathbb{R} $ thỏa mãn điều kiện:

$$f\left( {f(x + y)f(x - y)} \right) = x^2 - yf(y),\forall x,y \in \mathbb{R} $$.


Câu 3:
Cho $p$ là nguyên tố. Tìm tất cả các số nguyên dương $n$ sao cho với mọi số nguyên $x$, nếu $x^n-1$ chia hết cho $p$ thì $x^n-1$ cũng chia hết cho $p^2$.

Câu 4:
Cho hai hình tam giác $PAB$ và $PCD$ sao cho $ PA=PB,\ PC=PD$, $P, A, C $ và $P, B, D$ tương ứng thẳng hàng theo thứ tự.
Đường tròn $S_1$ đi qua $A, C$ cắt đường tròn $S_2$ đi qua $B, D$ tại hai điểm $X, Y$ phân biệt.
Chứng minh rằng tâm đường tròn ngoại tiếp của tam giác $PXY$ là trung điểm của đoạn thẳng nối tâm các đường tròn $S_1, S_2$


Câu 5.
Cho một quân cờ nằm tại gốc tọa độ trên mặt phẳng tọa độ, hai người $A, B$ chơi 1 trò chơi như sau:
Đầu tiên, $A$ đánh dấu vào một điểm trên lưới tọa độ, khác với điểm đang đặt quân cờ.
Sau đó $B$ di chuyển quân cờ từ điểm $(x,y)$ đến điểm $(x+1;y)$ hoặc $(x;y+1)$ $m$ lần $(1 \leq m \leq k)$ nhưng không được di chuyển quân cờ đến các điểm đánh dấu.
$A$ chiến thắng nếu $B$ không còn di chuyển được quân cờ. Tìm tất cả các số nguyên dương $k$ sao cho $A$ có thể chiến thắng sau một số hữu hạn lượt đi bất kể $B$ di chuyển quân cờ như thế nào.

Nguồn: AoPS

Dịch: Hoàng Ngọc Thế



img src=


#298833 Đôi chút về một lớp BĐT trong các kì thi ĐH

Gửi bởi Ban Biên Tập trong 10-02-2012 - 18:02

ĐÔI CHÚT VỀ MỘT LỚP BĐT TRONG

CÁC KÌ THI ĐẠI HỌC


Nguyễn Công Định - GV THPT Đầm Dơi - Cà Mau



Nội dung BĐT là một nội dung quan trọng trong các kì thi CĐ, ĐH, THCN. Nhận xét sơ bộ về các bài toán BĐT trong nhiều năm gần đây ta có thể chia thành hai loại:
1) Chứng minh BĐT bằng cách dựa vào hàm số.
2) Chứng minh BĐT dựa vào Cauchy.

Trong bài viết nhỏ này tôi sẽ trình bày nội dung thứ hai. Trước hết ta nhắc lại hai phát biểu đơn giản của Cauchy:
$(1)$: Cho $A, B$ không âm: $A + B \geq 2\sqrt {AB}$

$(2)$: Cho $A, B, C$ không âm: $A + B + C \geq 3\sqrt[3]{{ABC}}$

Từ $(1), (2)$ ta có thể thu được các BĐT thức không kém phần quan trọng sau:
$\begin{array}{l}
(3):\,\,AB \leq \left( {\dfrac{{A + B}}{2}} \right)^2 \\\\
(4):\,\,ABC \leq \left( {\dfrac{{A + B + C}}{3}} \right)^3 \\\\
(5):\,\,\dfrac{1}{A} + \dfrac{1}{B} \geq \dfrac{4}{{A + B}} \\\\
(6):\,\,\dfrac{1}{A} + \dfrac{1}{B} + \dfrac{1}{C} \geq \dfrac{9}{{A + B + C}} \\\\
(7):\,\,\left( {A + B} \right)^2 \leq 2\left( {A^2 + B^2 } \right) \\\\
(8):\,\,\left( {A + B + C} \right)^2 \leq 3\left( {A^2 + B^2 + C^2 } \right)\,... \\
\end{array}$

Học sinh có thể chứng minh các khẳng định trên tương đối đơn giản. Điểm chung từ $(1)$ đến $(8)$ là dấu “$=$” xảy ra khi và chỉ khi $A = B = C $.

Bây giờ ta giải một số bài toán cụ thể.

Dạng 1: Sử dụng trực tiếp từ (1) đến (10):

Bài 1:
Cho $a, b, c$ dương và $a + b + c = 3$. Chứng minh:
$$\sqrt[3]{a} + \sqrt[3]{b} + \sqrt[3]{c} \leq 3$$
Hướng dẫn:
Ta có:
$$\left. \begin{array}{l}
a + 1 + 1 \geq 3\sqrt[3]{a} \\
b + 1 + 1 \geq 3\sqrt[3]{b} \\
c + 1 + 1 \geq 3\sqrt[3]{c} \\
\end{array} \right\} \Rightarrow \,3\,VT\, \leq a + b + c + 6 = 9 \Rightarrow \,VT\, \leq 3.$$
Để tiện cho việc trình bày, từ bài tập này về sau nếu không có chú thích gì thêm thì ta xem $a, b, c$ dương.

Bài 2:
Cho $a + b + c = 1$. Chứng minh:
$$\dfrac{1}{{a + b}} + \dfrac{1}{{b + c}} + \dfrac{1}{{c + a}} > 4$$
Hướng dẫn:
(Sử dụng (6)).
$$VT \geq \dfrac{9}{a + b + b + c + c + a} = \dfrac{9}{2} > 4.$$
Bài 3:
Cho $a + b + c = 1$. Chứng minh:
$$\left( a + b \right)\left( b + c \right)\left( c + a \right) \leq \dfrac{8}{27}$$
Hướng dẫn:
(Sử dụng (3)).
$$VT \leq \left( \dfrac{a + b + b + c + c + a}{3} \right)^3 = \left( \dfrac{2}{3} \right)^3 = \dfrac{8}{27}.$$

Bài 4:
Cho $\dfrac{1}{{1 + a}} + \dfrac{1}{{1 + b}} + \dfrac{1}{{1 + c}} = 1.$ Chứng minh: $abc \geq 8.$
Hướng dẫn:
Với lưu ý:
$$1 - \dfrac{1}{{1 + a}} = \dfrac{a}{{1 + a}};\,1 - \dfrac{1}{{1 + b}} = \dfrac{b}{{1 + b}};\,1 - \dfrac{1}{{1 + c}} = \dfrac{c}{{1 + c}}$$
nên:
$$\dfrac{a}{{1 + a}} = \dfrac{1}{{1 + b}} + \dfrac{1}{{1 + c}} \geq \dfrac{2}{{\sqrt {\left( {1 + b} \right)\left( {1 + c} \right)} }}$$
$$\dfrac{b}{{1 + b}} = \dfrac{1}{{1 + c}} + \dfrac{1}{{1 + a}} \geq \dfrac{2}{{\sqrt {\left( {1 + c} \right)\left( {1 + a} \right)} }}$$
$$\dfrac{c}{{1 + c}} = \dfrac{1}{{1 + a}} + \dfrac{1}{{1 + b}} \geq \dfrac{2}{{\sqrt {\left( {1 + a} \right)\left( {1 + b} \right)} }}$$
Suy ra
$$\dfrac{{abc}}{{\left( {1 + a} \right)\left( {1 + b} \right)\left( {1 + c} \right)}} \geq \dfrac{8}{{\left( {1 + a} \right)\left( {1 + b} \right)\left( {1 + c} \right)}}$$
Từ đó, ta có đpcm.

Bài 5:
Cho $\dfrac{1}{{1 + a}} + \dfrac{1}{{1 + b}} + \dfrac{1}{{1 + c}} \geq 2$. Chứng minh: $abc \leq \dfrac{1}{8}.$
Hướng dẫn:
Với lưu ý:
$$\dfrac{1}{{1 + a}} \geq 1 - \dfrac{1}{{1 + b}} + 1 - \dfrac{1}{{1 + c}} = \dfrac{b}{{1 + b}} + \dfrac{c}{{1 + c}}$$
nên:
$$\dfrac{1}{{1 + a}} \geq \dfrac{b}{{1 + b}} + \dfrac{c}{{1 + c}} \geq \dfrac{{2\sqrt {bc} }}{{\sqrt {\left( {1 + b} \right)\left( {1 + c} \right)} }}$$
$$\dfrac{1}{{1 + b}} \geq \dfrac{{2\sqrt {ca} }}{{\sqrt {\left( {1 + c} \right)\left( {1 + a} \right)} }}$$
$$\dfrac{1}{{1 + c}} \geq \dfrac{{2\sqrt {ab} }}{{\sqrt {\left( {1 + a} \right)\left( {1 + b} \right)} }}$$
Từ đó
$$\dfrac{1}{{1 + a}}.\dfrac{1}{{1 + b}}.\dfrac{1}{{1 + c}} \geq \dfrac{{8abc}}{{\left( {1 + a} \right)\left( {1 + b} \right)\left( {1 + c} \right)}}$$
Suy ra đpcm.

Bài 6:
Cho $a + b + c \leq 1$. Chứng minh:
$$\left( {1 + \dfrac{1}{a}} \right)\left( {1 + \dfrac{1}{b}} \right)\left( {1 + \dfrac{1}{c}} \right) \geq 64.$$
Hướng dẫn:
Chú ý:
$$1 \geq a + b + c \geq 3\sqrt[3]{{abc}}\, \Rightarrow \dfrac{1}{{\sqrt[3]{{abc}}}} \geq 3
$$. Ta có:
$$\begin{matrix}
VT&= & 1 + \dfrac{1}{a} + \dfrac{1}{b} + \dfrac{1}{c} + \dfrac{1}{{ab}} + \dfrac{1}{{bc}} + \dfrac{1}{{ca}} + \dfrac{1}{{abc}}\\ \\
& \geq &1 + \dfrac{3}{{\sqrt[3]{{abc}}}} + \dfrac{3}{{\left( {\sqrt[3]{{abc}}} \right)^2 }} + \dfrac{1}{{\left( {\sqrt[3]{{abc}}} \right)^3 }} \\ \\
& \geq & \left( {1 + \dfrac{1}{{\sqrt[3]{{abc}}}}} \right)^3 \geq (1 + 3) ^4=64\\
\end{matrix}$$


Bài 7:
Cho $a + b + c \leq 1$. Chứng minh:
$$\dfrac{1}{{a^2 + 2bc}} + \dfrac{1}{{b^2 + 2ca}} + \dfrac{1}{{c^2 + 2ab}} \geq 9$.$
Hướng dẫn:
(Sử dụng (6)).
$$VT\, \geq \dfrac{9}{{a^2 + b^2 + c^2 + 2ab + 2bc + 2ca}} = \dfrac{9}{{\left( {a + b + c} \right)^2 }} \geq 9$$.
Bài 8:
Chứng minh:
$$\left( {a + \dfrac{1}{b}} \right)^2 + \left( {b + \dfrac{1}{c}} \right)^2 + \left( {c + \dfrac{1}{a}} \right)^2 \geq 12.$$
Hướng dẫn:
(Sử dụng (8)).
$$VT\, \geq \dfrac{1}{3}\left( {a + \dfrac{1}{b} + b + \dfrac{1}{c} + c + \dfrac{1}{a}} \right)^2 \geq \dfrac{1}{3}.6^2 = 12.$$
Bài 9:
Cho $\dfrac{1}{a} + \dfrac{1}{b} + \dfrac{1}{c} = 1$. Chứng minh:
$$\dfrac{1}{{2a + b + c}} + \dfrac{1}{{a + 2b + c}} + \dfrac{1}{{a + b + 2c}} \leq \dfrac{1}{4}$$.
Hướng dẫn:
(Sử dụng (5) hai lần):
$$\dfrac{4}{{2a + b + c}} \leq \dfrac{1}{{a + b}} + \dfrac{1}{{a + c}} \leq \dfrac{1}{4}\left( {\dfrac{1}{a} + \dfrac{1}{b} + \dfrac{1}{a} + \dfrac{1}{c}} \right)$$
$$\dfrac{4}{{a + 2b + c}} \leq \dfrac{1}{4}\left( {\dfrac{1}{a} + \dfrac{1}{b} + \dfrac{1}{b} + \dfrac{1}{c}} \right)$$
$$\dfrac{4}{{a + b + 2c}} \leq \dfrac{1}{4}\left( {\dfrac{1}{a} + \dfrac{1}{c} + \dfrac{1}{b} + \dfrac{1}{c}} \right)$$
$$4\,VT\, \leq \dfrac{1}{4}\left( {\dfrac{4}{a} + \dfrac{4}{b} + \dfrac{4}{c}} \right) = 1$$
Ta có đpcm.

Bài 10:
Cho $0 \leq a,b,c \leq 1$. Chứng minh:
$$\dfrac{a}{{b + c + 1}} + \dfrac{b}{{a + c + 1}} + \dfrac{c}{{a + b + 1}} + \left( {1 - a} \right)\left( {1 - b} \right)\left( {1 - c} \right) \leq 1.$$
Hướng dẫn:
Giả sử $a \leq b \leq c$. (Sử dụng (4)). Ta có:
$$\left( {a + b + 1} \right)\left( {1 - a} \right)\left( {1 - b} \right) \leq \left( {\dfrac{{a + b + 1 + 1 - a + 1 - b}}{3}} \right)^3 = 1 $$
Suy ra
$$\Rightarrow \left( {1 - a} \right)\left( {1 - b} \right)\left( {1 - c} \right) \leq \dfrac{{1 - c}}{{a + b + 1}}.$$
Nên:
$$VT\, \leq \dfrac{a}{{b + a + 1}} + \dfrac{b}{{a + b + 1}} + \dfrac{c}{{a + b + 1}} + \dfrac{{1 - c}}{{a + b + 1}} = 1.$$
Bài 11:
Cho $a + b + c =1$. Chứng minh:
$$\dfrac{a}{{1 + a}} + \dfrac{b}{{1 + b}} + \dfrac{c}{{1 + c}} \leq \dfrac{3}{4}.$$
Hướng dẫn:
Ta có: $$1 - \dfrac{a}{{1 + a}} + 1 - \dfrac{b}{{1 + b}} + 1 - \dfrac{c}{{1 + c}} = \dfrac{1}{{1 + a}} + \dfrac{1}{{1 + b}} + \dfrac{1}{{1 + c}} \geq \dfrac{9}{{3 + a + b + c}} = \dfrac{9}{4}.$$
Suy ra:
$$VT\, \leq 3 - \dfrac{9}{4} = \dfrac{3}{4}.$$

Bài 12:
Cho $a^2 + b^2 + c^2 = 1.$ Chứng minh:
$$\dfrac{1}{a} + \dfrac{1}{b} + \dfrac{1}{c} - \left( {a + b + c} \right) \geq 2\sqrt 3 .$$
Hướng dẫn:
(Sử dụng (8)), suy ra:
$$a + b + c \leq \sqrt 3 \Rightarrow - \left( {a + b + c} \right) \geq - \sqrt 3$$
(Sử dụng (6)), ta có:
$$\dfrac{1}{a} + \dfrac{1}{b} + \dfrac{1}{c} \geq \dfrac{9}{{a + b + c}} \geq \dfrac{9}{{\sqrt 3 }} = 3\sqrt 3$$.
Cộng hai BĐT cùng chiều trên ta có điều phải chứng minh.

Ta thấy chỉ việc sử dụng một vài BĐT ở dạng rất cơ bản như trên có thể giải quyết nhiều bài toán, tuy nhiên do khuôn khổ bài viết có hạn nên tôi không thể trình bày hết các bài tập dạng này được. Hẹn các bạn ở bài viết tiếp theo với dạng 2 là kĩ thuật tách ghép Cauchy.




#298687 Suy nghĩ về vai trò của toán học trong xã hội

Gửi bởi Ban Biên Tập trong 09-02-2012 - 09:40

Dù hơn hai ngàn năm nay toán học đã chứng tỏ mình như một đỉnh cao trí tuệ của con người, xâm nhập vào hầu hết các ngành khoa học và là nền tảng của nhiều lí thuyết khoa học quan trọng, nhưng với không ít người, vai trò của “nữ hoàng khoa học” trong đời sống xã hội vẫn còn là đối tượng tranh luận.

Trong cuộc tranh luận đó luôn có hai luồng. Một luồng đánh giá cao vai trò của toán học, trong khi luồng khác nghi ngờ vai trò của nó, thậm chí phủ định[1]. Thực tế này khiến chúng ta phải đặt lại câu hỏi: toán học có vai trò đến đâu trong xã hội chúng ta (trong khoa học tự nhiên vai trò của toán học không cần bàn cãi) và vì sao xã hội chưa thấy hết vai trò của toán học.

Vì sao xã hội không thấy hết vai trò của Toán học?

Từ thế kỷ mười chín trở về trước, một nhà toán học có thể vừa là nhà vật lí, nhà triết học hay nhà tự nhiên học. Sang thế kỷ hai mươi khi toán học đã trở thành một ngành độc lập, phần lớn các nhà toán học cũng trở thành những nhà toán học thuần túy, xa rời và ít quan tâm đến những vấn đề thực tế. Phần lớn thời gian của họ được dành để giải quyết các vấn đề phát triển nội tại của toán học vốn ngày càng phức tạp hay các vấn đề khoa học hàn lâm khác theo kiểu “toán học vị toán học”, tương tự như trào lưu “nghệ thuật vị nghệ thuật”. Hiện tượng này thực ra không có gì lạ, vì như lời một học giả, mỗi khi một sự vật nào đó đã tích lũy được một lượng tri thức nhất định, nó sẽ bắt đầu sống đời sống riêng của nó, ngoại trừ một phần tri thức sẽ ra phục vụ bên ngoài còn phần lớn là đời sống nội tại của chính nó. Toán học cũng không nằm ngoài qui luật này. Nội tại của nó phong phú tới mức “thậm chí ngay một bộ phận nào đó của toán học thuần túy đã rộng lớn đến mức vượt qua khả năng thấu hiểu của con người”[2].

Thế nên dù vai trò của toán học trong các ngành khoa học tự nhiên là vô cùng to lớn, trong con mắt xã hội hình ảnh các nhà toán học cùng những lý thuyết toán học của họ trở nên xa lạ. Năm 1980 tại Warszawa tác giả bài viết này đã chứng kiến cuộc chia tay với Kazimierz Kuradowski, nhà toán học lớn của thế giới, một chuyên gia hàng đầu về Topo: Chỉ một thông báo nhỏ trên báo Đảng và một đoàn người không đông gồm đồng nghiệp và học trò đưa tiến ông đến nới an nghỉ cuối cùng. Cùng thời gian đó có hàng vạn người hâm mộ theo sau đám tang của một vận động viên thể thao nổi tiếng. Những người làm toán chắc không tránh khỏi chạnh lòng dẫu họ rất hiểu nghề nghiệp mà mình đã lựa chọn (như cụ Nguyễn Du đã từng viết trong Truyện Kiều: “Đã mang lấy nghiệp vào thân, cũng đừng trách lẫn trời gần trời xa”).

Sự thật là toán học có vai trò rất to lớn trong đời sống thường ngày nhưng không dễ nhìn thấy. Nó có mặt trong các thiết bị được sử dụng rộng rãi nhưng thường bị che lấp bởi công nghệ. Liệu có bao nhiêu khách hàng thuê bao điện thoại biết được để mạng điện thoại vận hành thông suốt có sự đóng góp không nhỏ của thuật toán đơn hình - một thuật toán cơ bản của lí thuyết qui hoạch toán học. Hàng loạt các thiết bị gia dụng thông minh ngày nay được tích hợp các phương pháp của logic mờ. Những người làm công ăn lương vẫn nhận tiền qua các máy ATM nhưng mấy ai biết nếu không có các thuật toán an toàn trong đó thì số tiền của họ sẽ không cánh mà chui vào túi của đạo chích. Và đó cũng chỉ là một số ví dụ đơn cử.

Nhiều tri thức toán học, ngay cả toán học đơn giản ở bậc phổ thông, có thể ứng dựng hiệu quả vào đời sống nhưng đòi hỏi những kĩ năng nhất định và một thói quen nhất định. Trang bị những kĩ năng này là công việc của nhà trường và sự rèn luyện của bản thân mỗi người. Nhưng trên thực tế, rất ít người, kể cả những người có học vấn tương đối, thực hiện những kỹ năng này. Không chỉ ở những nước còn lạc hậu mà ngay tại những nước tiên tiến như Hoa Kỳ, theo nhận xét của Andrei Okunkov, nhà toán học Nga đoạt giải Fields, giáo sư Đại học Princeton, người Mỹ đều mong muốn trở nên giàu có khi về già nhưng không mấy ai biết vận dụng một số kĩ năng của lí thuyết xác suất khả dĩ có thể giúp họ đưa ra những quyết định có lợi cho việc thực hiện giấc mơ của mình [2].

Vài suy nghĩ về vai trò của toán học trong xã hội

Hơn một trăm năm trước Karl Marx đã nói rằng một ngành khoa học chỉ trở nên hoàn thiện khi nó sử dụng được ngành khoa học định lượng-đó là toán học. Lịch sử phát triển các ngành khoa học tự nhiên đã hoàn toàn khẳng định luận điểm này của Marx. Nhưng luận điểm đó còn đúng cả với nhiều lĩnh vực xã hội.

Được thôi thúc bởi khát vọng tìm kiếm và sáng tạo, các nhà toán học đã không dừng lại ở các ngành khoa học tự nhiên mà chuyển sang cả các lĩnh vực xã hội. Trong nửa đầu thế kỷ hai mươi họ đã cho ra đời không ít công cụ toán học có thể áp dụng để phân tích bản chất các quá trình xã hội: các phương pháp thống kê xã hội, lí thuyết toán học các xung đột và hợp tác(lí thuyết trò chơi), các mô hình toán học trong kinh tế, phương pháp phân tích hệ thống, lí thuyết các hệ động lực. Một số nhà toán học đã giành được giải Nobel, một giải thưởng khoa học danh giá vốn không dành cho các nhà toán học, như Kantorovich - Nhà toán học Nga, “vì những đóng góp vào lí thuyết phân bố tối ưu tài nguyên” và John Nash - nhà toán học Mỹ, “vì các công trình về lí thuyết trò chơi”.

Từ đầu thập kỷ bảy mươi của thế kỷ trước sự ra đời của máy tính điện tử đã tạo ra một bước ngoặt mới cho việc áp dụng toán học vào xã hội, và ở chừng mực nào có thể nói từ đây toán học cũng đã trở thành một ngành khoa học thực nghiệm giống như vật lí, hóa học, sinh học và một số ngành khác. Nghĩa là ban đầu các quá trình xã hội được mô hình hóa dưới dạng ngôn ngữ toán học (gọi là mô hình toán học-hệ thống các tương quan toán học mô tả dưới dạng thu gọn quá trình xã hội), sau đó chúng được chạy trên máy tính điện tử và có thể được thử đi thử lại nhiều lần. Trên cơ sở đó, người ta đã thu được nhiều kết quả quan trọng.

Các nhà toán học còn tiến xa hơn, họ đã không dừng lại ở việc mô phỏng các quá trình xã hội ở qui mô nhỏ, vừa, mà thậm chí còn mô phỏng cả những vấn đề ở tầm hành tinh. Từ đây đã ra đời một lĩnh vực liên ngành rộng lớn: mô hình hóa toàn cầu (global modeling) và nhiều hướng mới trong khoa học: lí thuyết toán học về phát triển, lí thuyết các hệ sinh thái, lí thuyết quyết định v.v. Qua đó con người đã thu được rất nhiều thành tựu cho phép phát hiện ra bản chất của các quá trình chính trị-xã hội.

Toán học không chỉ góp phần vào phân tích và khám phá những bí mật của các quá trình xã hội, toán học còn là bộ phận cấu thành không thể thiếu của những sản phẩm phục vụ đời sống hằng ngày: các hàm băm toán học (hash functions) trong các cấu trúc an ninh của hệ điều hành máy tính, các thuật toán bảo vệ dữ liệu cá nhân và xác thực danh tính trong các thẻ giao dịch tài chính, ngân hàng, các thuật toán tạo chữ kí điện tử thay thế chữ kí tay, tổ hợp các thuật toán trong chứng thư điện tử được sử dụng trong giao dịch điện tử, công nghệ toán học mờ (Fuzzy Mathematics) trong các thiết bị điều khiển và các thiết bị gia dụng. Có vô vàn những ví dụ khác mà người ta có thể kể ra.

Những hạn chế

Tuy nhiên theo ý kiến của nhiều nhà khoa học, những thành tựu của việc áp dụng toán học vào khoa học xã hội còn rất hạn chế, và rất khiêm tốn so với những thành tựu của nó trong lĩnh vực khoa học tự nhiên. Ví như trong kinh tế học, nơi toán học được áp dụng sớm nhất và đạt nhiều thành tựu nhất, nhưng trên thực tế vẫn chưa thể phán ánh hết được những hiện tượng quan trọng nhất của kinh tế [5].

Việc ứng dụng toán học vào xã hội có những đặc thù riêng là nguyên nhân của hạn chế nói trên. Muốn áp dụng toán học nhà toán học phải xây dựng được mô hình toán học - tập hợp các quan hệ toán học phán ảnh những khía cạnh định lượng của các quá trình xã hội. Đây được coi là khâu khó nhất; thứ đến mới là công việc giải mô hình, tức là giải các vấn đề toán học trên đó. Nhưng khi xây dựng mô hình các nhà toán học phải làm việc với đối tượng có bản chất hoàn khác với các đối tượng của tự nhiên, tính bất định cao hơn, nhiều yếu tố ngẫu nhiên hơn, và nếu mô hình muốn tính đến cả hành vi xã hội bao gồm hành vi của tổ chức và con người thì có lẽ như Andrei Okunkov đã nói “ở đây toán học chạm vào ranh giới của cái không thể ”[2].

Hơn nữa muốn xây dựng được mô hình toán học phản ánh được các quá trình xã hội đòi hỏi các nhà toán học phải xâm nhập sâu vào các quá trình đó và hợp tác chặt chẽ với các nhà nghiên cứu xã hội. Điều này không phải nhà toán học nào cũng sẵn sàng hoặc không phải lúc nào cũng có điều kiện. Nikita Moiseev - người có cống hiến to lớn trong ứng dụng toán học vào xã hội, cho biết để xây dựng các mô hình kinh tế ông đã phải vừa học vừa làm trong hơn mười năm. Paul Embrechts, người đã cảnh báo về khủng hoảng tài chính thế năm 2008, là một nhà toán học đã dành nhiều năm làm việc cho giới tài chính.

Trong sự hợp tác giữa toán học và các ngành xã hội, tất nhiên sẽ có sự đụng độ giữa hai lối tiếp cận, hai kiểu tư duy: với các nhà toán học là sự chặt chẽ, chính xác; với các nhà xã hội là tính khuynh hướng, sự ước lệ trong ngôn ngữ. Vì vậy chỉ có thể xây dựng được mô hình toán học cho các quá trình xã hội khi biết kết hợp các phương pháp của toán học với kĩ thuật phân tích của xã hội học, một việc mà trong thực tế không phải lúc nào cũng làm được. Hơn nữa kết quả thu được bao giờ cũng là phương án thỏa hiệp. Do đó dù mô hình toán học tốt đến đâu cũng chỉ là phương án thu gọn thực tiễn, và chỉ phán ánh một số khía cạnh nào đó của sự vật, khiến những kết luận mà chúng đem lại thường là đối tượng gây tranh cãi và không được xem là “các định lí về xã hội”. Một đặc điểm nổi bật nữa của một mô hình toán học trong xã hội là việc thử nghiệm trên thực tế đòi hỏi đầu tư tốn kém nhân lực và vật lực, hơn nữa lại không thể lặp đi lặp lại nhiều lần như ở các thí nghiệm vật lí hoặc hóa học.

Hãy đo, dù không phải lúc nào cũng đo được

Quá trình sản xuất và đời sống ngày càng được tự động hóa thì xã hội ngày càng trở nên nhân tạo và vai trò của toán học ngày càng lớn. Dĩ nhiên không phải hiện tượng xã hội nào cũng có thể được mô phỏng qua mô hình toán học, nhưng không gì có thể ngăn cản sự tò mò, sáng tạo và khát khao chinh phục những vùng đất mới của các nhà toán học. Hàng ngàn năm trước nhà triết học Socrates đã nói “Hãy đo, hãy đo, dù không phải lúc nào cũng đo được”. Các nhà toán học một cách ý thức hay vô thức hình như đang đi theo lời kêu gọi đó. Họ trở thành một lực lượng đông đảo, có đóng góp lan tỏa rộng khắp trong đời sống. Toán học ngày nay đã trở thành một nghề nghiệp, đảm bảo việc làm cho hàng chục vạn người làm toán trên thế giới, góp phần phát triển và ổn định xã hội. Chỉ ngần ấy đủ cho thấy vai trò không thể phủ định của toán học trong xã hội.

---------------------

Tài liệu tham khảo

[1] Người làm toán giỏi: Rất lãng phí –Vietbao.vn 21/2/2006

[2] Anerei Okunkov nói về toán học- Tia sáng 20/11/2011

[3] Nguyễn Tiến Dũng: Nhà toán học đã cảnh báo khủng hoảng tài chính.Zung.Zetamu.net 8/10/2011

[4] John Forber Nash: Wikipedia

[5] Nikita Moiseev- Chia tay với sự giản đơn (Bản tiếng nga).Moskva 1998

[6] Dennis Meadows,Donella Meadows,Jorgen Renders: The Limits to growth

[7] Jorgen Stig Norgard, John Peet: The History Limit to Growth

[8] N.Moiseev, V Alexandrov, A. Tarko: Con người và sinh quyển (Tiếng Nga)-Moskva 1985

Theo Tia Sáng


#298463 sử dụng ma trận nhận thức để định lượng thời gian ôn tốt nghiệp THPT

Gửi bởi Ban Biên Tập trong 07-02-2012 - 12:40

SỬ DỤNG MA TRẬN NHẬN THỨC ĐỂ ĐỊNH LƯỢNG

THỜI GIAN ÔN THI TÔT NGHIỆP THPT


Hoàng Ngọc Thế - GV THPT Pác Khuông - Lạng Sơn


Chỉ còn vài tháng nữa là đến kì thi Tốt nghiệp THPT. Thời điểm này, nhiều trường THPT đang tiến hành lập kế hoạch ôn thi. Trong bài viết này, tôi trình bày ý tưởng sử dụng ma trận nhận thức để định lượng thời gian ôn tốt nghiệp THPT.

Trước hết, xin nhắc lại quy trình biên soạn 1 đề kiểm tra. Biên soạn một đề kiểm tra cần thực hiện theo quy trình sau:
Bước 1. Xác định mục đích, yêu cầu đề kiểm tra
Bước 2. Xác định mục tiêu dạy học và hình thức đề kiểm tra
Bước 3. Thiết lập ma trận đề kiểm tra
Bước 4. Biên soạn câu hỏi, bài tập theo ma trận đề
Bước 5. Xây dựng hướng dẫn chấm (đáp án) và biểu điểm
Bước 6. Xem xét lại việc biên soạn đề kiểm tra

Ta sẽ tìm hiểu bước 3. Để xây dựng ma trận đề kiểm tra, người ta cần xây dựng một ma trận nhận thức. Cách xây dựng ma trận nhận thức được nhắc lại dưới đây và tôi sẽ lấy ví dụ đối với chương I của Giải tích 12.

1) Lập (theo cột) danh sách các nội dung - chủ đề hay mạch kiến thức kĩ năng mà bạn cho là mục tiêu học tập phải đạt của học sinh theo Chuẩn xét đến thời điểm thực hiện Chương trình Giáo dục.
b1.PNG

2. Xác định tầm quan trọng của mỗi chủ đề hoặc mạch kiến thức kĩ năng của Chuẩn trong tổng thể khối nội dung chọn qua việc lượng hóa theo tỉ lệ % (tùy theo người thiết kế xác định về tầm quan trọng của chủ đề, mạch kiến thức kĩ năng hoặc về thời lượng tương ứng học sinh tiếp thu nó trong tổng thể khối chọn); Tổng các tỉ lệ % lượng hóa phải bẳng 100%
b3.PNG

3. Xác định trọng số từ 1 đến 4 cho mức độ nhận thức của mỗi chủ đề, mạch kiến thức kĩ năng trong Chuẩn tùy theo người thiết kế xác định xét đến thời điểm thực hiện Chương trình Giáo dục, (mức đầu ra của quá trình nhận thức xét đến thời điểm thực hiện chương trình), quan trọng là phải phù hợp với đối tượng học sinh.
b4.PNG


4. Nhân tỉ lệ % lượng hóa mức độ cơ bản, trọng tâm của mỗi chủ đề, mạch kiến thức kĩ năng với trọng số của nó để xác định điểm số của mỗi chủ đề hay mạch kiến thức kĩ năng


5. Cộng số điểm của tất cả các chủ đề, mạch kiến thức kĩ năng để xác định tổng số điểm của ma trận.
- Tổng số điểm của ma trận cao nhất là 400 điểm và thấp nhất là 100 điểm.
- Nếu tổng số điểm là 400 thì đó phương án lựa chọn tốt nhất dựa theo chuẩn chọn nội dung và mức nhận thức cho dạy, kiểm tra đánh giá.
- Nếu tổng số điểm là 250 = (400 + 100):2, thì đó phương án lựa chọn trung bình dựa theo chuẩn chọn nội dung và mức nhận thức cho dạy, kiểm tra đánh giá.
- Nếu tổng số điểm là 100 thì đó phương án lựa chọn yếu kém dựa theo chuẩn chọn nội dung và mức nhận thức cho dạy, kiểm tra đánh giá.
b5.PNG

6. Căn cứ vào tỉ lệ của điểm số ở mỗi chủ đề trên tổng điêm, ta quy ra điểm của nội dung chủ đề đó trong đề kiểm tra
b6.PNG

Như vậy là xuất phát từ số tiết học (cột C), đối tượng học sinh (cột E) mà ta đã xác định được điểm số (cột G) của từng chủ đề trong đề kiểm tra.

Vấn đề bây giờ là nếu ta biết trước điểm số (cột G) và đối tượng học sinh (cột E) ta có quy ra số tiết ôn tập (cột C) được không? Câu trả lời là có.

Chúng ta sẽ định lượng thời gian ôn thi cho từng chủ đề. Đầu tiên, ta nhắc lại các chủ đề theo cấu trúc đề thi của Bộ GD&ĐT.
c2.PNG

Tiếp theo, căn cứ vào đối tượng học sinh, ta đánh giá sức học của học sinh đối với từng chủ đề. Điểm đánh giá chính là trọng số (cột E). Chủ đề nào khó (đối với học sinh) thì cho điểm cao, chủ đề nào dễ (đối với học sinh) thì cho điểm thấp, chẳng hạn:
c3.PNG

Theo trên, ta có

Tổng điểm (Cột F) = Trọng số (cột E) x tầm quan trọng (cột D)

Vậy

Tầm quan trọng (cột D) = Tổng điểm (Cột F) ÷ Trọng số (cột E)

Chú ý rằng điểm số (cột G) và tổng điểm (cột F) tương ứng tỉ lệ. Do đó ta sẽ tìm tầm quan trọng của mỗi chủ đề bằng cách sau:

Chỉ số quan trọng (cột D’) = Điểm số (Cột G) ÷ Trọng số (cột E)

c4.PNG

Tầm quan trọng (cột D) chính là tỉ lệ phần trăm của chỉ số quan trọng (cột D’) đối với tổng các chỉ số (3.583333).
c5.PNG

Bây giờ ta cần quan tâm đến số tiết mà nhà trường bố trí cho môn Toán. Chẳng hạn: 58 tiết. Ta có:

Số tiết (cột C) = Tầm quan trọng (cột D) x Tổng số tiết (58) ÷ 100

Ta làm tròn thành số nguyên.
c6.PNG

Như vậy ta đã xác định được số tiết ôn đối với mỗi chủ đề để phù hợp với đối tượng học sinh.
Nhận xét: Phương pháp này đảm bảo rằng chủ đề nào dễ mà lại được nhiều điểm sẽ được ôn nhiều, chủ đề khó mà được ít điểm sẽ được ôn ít.
Bài tập:
1) Bạn hãy tự xác định thời lượng ôn tập sao cho phù hợp với học sinh của mình và tổng số tiết ôn của trường.
2) Nêu thêm các vấn đề khác


#298154 Đề cương thi Olympic Toán Sinh viên năm 2012

Gửi bởi Ban Biên Tập trong 05-02-2012 - 12:39

Kỳ thi Olympic Toán Sinh viên lần thứ 20 (2012) sẽ được tổ chức tại Phú Yên từ 09 -15/4/2012.

Hình đã gửi


Nội dung chương trình thi Olympic SV 2012 như sau:

ĐỀ CƯƠNG ÔN TẬP MÔN ĐẠI SÔ

Phần I: Đại số tuyến tính

1. Không gian véc tơ
- Định nghĩa, không gian con và các ví dụ liên quan tới giải tích
- Hệ sinh, hệ độc lập tuyến tính, cơ sở
- Các tính chất cơ bản của ánh xạ tuyến tính và mối liên hệ với ma trận biểu diễn

2. Giá trị riêng và véc tơ riêng của ánh xạ tuyến tính và của ma trận: định nghĩa, các tính chất cơ bản, cách tìm.
3. Ma trận, định thức
- Ma trận (thực, phức), các phép toán của ma trận và một số tính chất.
- Định thức: định nghĩa (quy nạp theo cấp n và theo phép thế), định lý Laplace, tính chất của định thức, các phương pháp tính định thức.
- Ma trận nghịch đảo, các phương pháp tìm ma trận nghịch đảo (theo phần bù đại số và biến đổi sơ cấp).
- Hạng của ma trận, cách tính hạng của ma trận.
- Ma trận đồng dạng và tính chéo hóa của ma trận.
- Phương trình ma trận. Đa thức đặc trưng, đa thức tối tiểu và Định lí Hamilton-Cayley.
- Một số dạng ma trận đặc biệt: ma trận Vandermonde, ma trận đối xứng, ma trận phản đối xứng, ma trận Hermite, ma trận trực giao.

4. Hệ phương trình tuyến tính.
- Dạng tổng quát và dạng ma trận của hệ phương trình tuyến tính. Hệ Cramer.
- Định lí Kronecker-Capelli.
- Phương pháp Gauss, phương pháp Gauss-Jordan
- Nghiệm riêng và nghiệm tổng quát của hệ phương trình tuyến tính. Không gian nghiệm của hệ phương trình tuyến tính thuần nhất.

Phần II: Đa thức
- Các phép toán của đa thức, phân tích một đa thức thành nhân tử, ước chung lớn nhất của 2 đa thức, hai đa thức nguyên tố cùng nhau.
- Nghiệm của đa thức: định lí Bezout, lược đồ Horner, định lí Viet, biên của nghiệm, quy tắc dấu Descartes.
- Đa thức dương, công thức Taylo
- Bài toán xác định đa thức (phương pháp hệ số bất định, các phương trình xác định đa thức…)

ĐỀ CƯƠNG ÔN TẬP MÔN GIẢI TÍCH
1. Dãy số :
- Dãy hội tụ, dãy đơn điệu, dãy bị chặn. Giới hạn vô cùng
- Các tính chất và các phép toán về dãy hội tụ.
- Tìm giới hạn của các dãy số.
- Phương trình và bất phương trình sai phân
2. Hàm số:
- Định nghĩa hàm số, miền xác định, miền giá trị, hàm đơn điệu, hàm bị chặn, hàm tuần hoàn, hàm chẵn, hàm lẻ, hàm ngược.
- Giới hạn hàm số.
- Sự liên tục của hàm số, các tính chất của hàm liên tục.
- Phương trình hàm, bất phương trình hàm.

3. Phép tính vi phân hàm một biến:
- Định nghĩa đạo hàm, hàm khả vi và các phép toán về đạo hàm.
- Các định lý: Fermat, Rolle, Lagrange, Cauchy, L’Hospital.
- Công thức Taylor, Maclaurin của hàm số.
- Cực trị, GTLN, GTNN của hàm số.
- Phương trình hàm trên lớp hàm khả vi.

4. Phép tính tích phân hàm một biến.
- Nguyên hàm và tích phân bất định.
- Các phương pháp tính tích phân bất định.
- Tích phân các hàm hữu tỷ, vô tỷ, hàm lượng giác.
- Hàm khả tính và tích phân xác định.
- Các phương pháp tính tích phân xác định.
- Tích phân có cận thay đổi.
- Định lý về giá trị trung bình của tích phân.
- Bất đẳng thức tích phân.

Theo vnmath.com